Posts

DAILY CURRENT AFFAIRS IAS | UPSC Prelims and Mains Exam – 15th May 2024

Archives (PRELIMS & MAINS Focus)   CHABAHAR PORT Syllabus Prelims & Mains – Current Event , GS 2 Context: India and Iran signed a 10-year contract on Monday for the operation of a terminal at the strategically important Chabahar port in Iran. Background:- The long-term bilateral contract was signed between Indian Ports Global Limited (IPGL) and the Port & Maritime Organisation (PMO) of Iran, enabling the operation of the Shahid Beheshti terminal at the Chabahar Port for 10 years.The pact replaces one-year contracts that were being signed to keep the port operational until now. About CHABAHAR PORT Chabahar is a deep water port in Iran’s Sistan-Baluchistan province. It is the Iranian port that is the closest to India, and is located in the open sea, providing easy and secure access for large cargo ships. History and Progress Modern Chabahar came into being in the 1970s, and Tehran realised the strategic importance of the port during the Iran-Iraq war of the 1980s. In January 2003, President Khatami and then Prime Minister Atal Bihari Vajpayee signed off on an ambitious roadmap of strategic cooperation. Among the key projects the two countries agreed on was Chabahar, which held the potential to link South Asia with the Persian Gulf, Afghanistan, Central Asia, and Europe. The timelines for the project were undone by India’s growing relationship with the United States. The US, which declared Iran as one of the “axis of evil” along with Iraq and North Korea, pushed New Delhi to abandon its strategic relationship with Tehran, and the Chabahar project became a casualty. While India spent about $100 million to construct a 218-km road from Delaram in western Afghanistan to Zaranj on the Iran-Afghan border to link with Chabahar, the port project itself progressed at a glacial pace. IPGL (India Ports Global Ltd) has been operating Chabahar port through its wholly owned subsidiary, India Ports Global Chabahar Free Zone (IPGCFZ), since December 24, 2018. The port has handled more than 90,000 twenty-foot-equivalent units (TEUs) of container traffic and more than 8.4 million metric tonnes (MMT) of bulk and general cargo since then. Chabahar and INSTC To better realise Chabahar ports potential, the development of the port must be integrated with the larger connectivity project of the International North South Transport Corridor (INSTC). The INSTC, which was initiated by Russia, India, and Iran, is a multi-modal transportation route envisaged to link the Indian Ocean and Persian Gulf to the Caspian Sea via Iran, and onward to northern Europe via St Petersburg in Russia. Source:Indian Express XENOTRANSPLANTATION Syllabus Prelims – Current Event Context: The first recipient of a modified pig kidney transplant passed away on Saturday (May 11), around two months after the surgery was carried out. Background: Richard Slayman got the first modified pig kidney transplant in March this year. Key Takeaways Xenotransplantation, derived from the Greek word “xenos” meaning “foreign” or “strange”, is the transplantation of living cells, tissues, or organs from one species to another. The transplanted cells, tissues, or organs are referred to as xenografts or xenotransplants. This process is contrasted with allotransplantation (the transplantation from one individual to another of the same species with a different genotype), syngeneic transplantation or isotransplantation (transplants between two genetically identical individuals of the same species), and autotransplantation (transplants from one part of the body to another in the same person) Xenotransplantation involving the heart was first tried in humans in the 1980s. The need for such a procedure was felt because of the significant gap between the number of transplantations needed by patients and the availability of donor organs. It offers a potential treatment for end-stage organ failure. A report from the Harvard Medical School, whose physicians were involved in Slayman’s operation, said 69 genomic edits were made to the pig kidney in his case. The gene editing technology CRISPR-Cas9 was employed to “Remove certain pig genes that produce sugars with antibodies our immune systems react to” and “Add certain human genes to improve the kidney’s compatibility with humans.” Why are pigs often used for xenotransplantation? Pig heart valves have been used for replacing damaged valves in humans for over 50 years now. The pig’s anatomical and physiological parameters are similar to that of humans, and the breeding of pigs in farms is widespread and cost-effective. Also, many varieties of pig breeds are farmed, which provides an opportunity for the size of the harvested organs to be matched with the specific needs of the human recipient. In January 2022, the first xenotransplantation of a genetically-modified pig heart was done. However, the patient passed away after two months due to a range of factors, including being tainted with a latent virus in the pig heart, which may have contributed to the dysfunction of the transplant. Source: Indian Express 55 Cancri e Syllabus Prelims – Current Event Context: A thick atmosphere has been detected around a planet that’s twice as big as Earth in a nearby solar system, researchers reported Wednesday.The so-called super Earth — known as 55 Cancri e — is among the few rocky planets outside our solar system with a significant atmosphere, wrapped a blanket of carbon dioxide and carbon monoxide. Background: Super Earth refers to a planet’s size — bigger than Earth but smaller than Neptune. The boiling temperatures on this planet — which can reach as hot as 4,200 degrees Fahrenheit – mean that it is unlikely to host life. About 55 Cancri e : 55 Cancri e, also known as Janssen, is an exoplanet that orbits a Sun-like host star, 55 Cancri A. The planet is located in our Milky Way galaxy about 41 light-years from Earth, in the constellation Cancer.A light year is the distance light travels in a year, 5.9 trillion miles (9.5 trillion km). Four other planets, all gas giants, are known to orbit its host star. It was discovered on August 30, 2004. It was the first super-Earth discovered around a main sequence star. The mass of the exoplanet is about eight Earth masses and its diameter is about twice that of the Earth. It is the innermost planet in its planetary system, taking less than 18 hours to complete an orbit. Temperature: Due to its proximity to its star, 55 Cancri e is extremely hot, with temperatures on the day side exceeding 3,000 Kelvin. Infrared observations using two instruments aboard the James Webb Space Telescope indicated the presence of a substantial – if inhospitable – atmosphere, perhaps continuously replenished by gases released from a vast ocean of magma. Being so close to its star, any atmosphere should be stripped away by stellar irradiation and winds. But gases dissolved in the vast lava ocean thought to cover the planet may keep bubbling up to replenish the atmosphere All of the previous exoplanets found to have atmospheres were gaseous planets, not rocky ones. As Webb pushes the frontiers of exoplanet exploration, the discovery of a rocky one with an atmosphere represents progress. What is an exoplanet? An exoplanet is any planet beyond our solar system. Most orbit other stars, but free-floating exoplanets, called rogue planets, orbit the galactic center and are untethered to any star. Source: Hindu Deepfake Syllabus Prelims – Current Event Context: Deepfakes has emerged as a major concern during election season. Background: AI-powered misinformation campaigns can influence voter behaviour by spreading false narratives or amplifying divisive content. About Deepfake : Deepfakes are a type of synthetic media where one person’s likeness is convincingly replaced with that of another. The term “deepfake” is a portmanteau of “deep learning” and “fake” A deepfake is an artificial image, audio or video generated by a special kind of machine learning called “deep” learning (hence the name) Deepfake technology leverages tools and techniques from machine learning and artificial intelligence, including facial recognition algorithms and artificial neural networks such as variational autoencoders (VAEs) and generative adversarial networks (GANs). It is used to manipulate videos, images, and audios. This technology can be used to generate fake news and commit financial fraud among other wrongdoings.It overlays a digital composite over an already-existing video, picture, or audio. Deepfake technology can seamlessly stitch anyone in the world into a video or photo they never actually participated in. How does deepfake technology work? The technology involves modifying or creating images and videos using a machine learning technique called generative adversarial network (GAN). The AI-driven software detects and learns the subjects’ movements and facial expressions from the source material and then duplicates these in another video or image. To ensure that the deepfake created is as close to real as possible, creators use a large database of source images. This is why more deepfake videos are created of public figures, celebrities and politicians. The dataset is then used by one software to create a fake video, while a second software is used to detect signs of forgery in it. Through the collaborative work of the two software, the fake video is rendered until the second software package can no longer detect the forgery. This is known as “unsupervised learning”, when machine-language models teach themselves. The method makes it difficult for other software to identify deepfakes. Examples: In January 2024, during the New Hampshire primary of the Democratic Party in the US, a robocall mimicking President Joe Biden’s voice falsely advised voters not to participate, claiming it would affect their eligibility for the general election. In Slovakia, an AI-generated voice, mimicking that of a liberal candidate, discussing plans to raise alcohol prices and rig the election was widely circulated on Facebook. In Bangladesh, deepfake videos of opposition politicians Rumin Farhana in a bikini and Nipun Roy in a swimming pool surfaced on social media ahead of the national elections. Source: Hindu INTERACTIVE VOICE RESPONSE SYSTEM (IVRS) Syllabus Prelims – Science Context: Recently, voters are receiving Interactive Voice Response System (IVRS) calls every day and they want the Election Commission of India (ECI) should order an inquiry as to how the service providers of various telecom firms shared our mobile phone numbers with the political parties. Background: The calls violate the Telecom Regulatory Authority of India (TRAI) Act, 1997. Lakhs of voters are affected daily by these IVRS calls. It’s essential for the ECI to address this issue promptly to protect voters’ privacy and ensure a fair electoral process. About INTERACTIVE VOICE RESPONSE SYSTEM (IVRS) : An Interactive Voice Response System (IVRS) is an automated telephony technology that allows a computer to interact with humans through the use of voice and DTMF tones input via a keypad. IVRS is commonly used in call centers to manage large volumes of calls and provide callers with self-service options. The benefits of using IVRS include improved customer service, reduced wait times, and lower operational costs for businesses. Modern IVRS can integrate with various networks, including public switched telephone networks (PSTN) and voice over IP (VoIP), and support multiple applications written in VoiceXML. Political Campaigns and IVRS: During elections, political parties use IVRS to reach out to voters. Voters receive pre-recorded messages from candidates or party leaders, urging them to vote or highlighting campaign promises. These messages can be informative or persuasive, depending on the party’s strategy. Source: Hindu ROAD SAFETY Syllabus Mains – GS 2 Context: In a bid to enhance road safety awareness across Uttar Pradesh, it was announced that the state government had earmarked Rs 100 million for the Transport Corporation. Background: For years, India has been trying to combat road accidents. With the world’s second-largest population and a booming vehicle market, the country faces a constant battle against devastating accidents. Factors responsible for Road Accidents: Sometimes highways come up haphazardly across the roads, and street design permits speeding, which leads to road safety concerns. The problem is further aggravated due to inadequate signage, road markings or incorrect location, and poor construction of speed breakers. Ongoing construction works astride roads restrict the availability of restricted space to the road user. Improper road markings, lack of traffic control, etc. at such sites further increase the vulnerability Heavy rain, dense fog, and hail storms reduce visibility and make the road surface slippery thus posing serious risks to the road users. Sidewalks not being available for pedestrian use who then move on the roads thereby becoming vulnerable to accidents. Old vehicles are relatively more prone to breakdown and malfunction. It can result in the bursting of tyres, road collapse, wearing out of brakes, overturning of the vehicle, etc. Violation of traffic rules, non-use of safety devices (helmets and seat belts), triple riding, overloading, distracted driving, road rage, etc. Suboptimal training and testing, and lack of coordination among different agencies. Indian Initiatives Related to Road Safety: Motor Vehicles Amendment Act of 2019 hikes the penalties for traffic violations, defective vehicles, juvenile driving, etc. It provides for a Motor Vehicle Accident Fund, which would provide compulsory insurance coverage to all road users in India for certain types of accidents. It also provides for a National Road Safety Board, to be created by the Central Government. The Carriage by Road Act of 2007 provides for the regulation of common carriers, limiting their liability and declaration of the value of goods delivered to them to determine their liability for loss of, or damage to, such goods occasioned by the negligence or criminal acts of themselves, their servants or agents and for matters connected therewith or incidental thereto. The Control of National Highways (Land and Traffic) Act of 2000 provides for the control of land within the National Highways, right of way, and traffic moving on the National Highways and also for removal of unauthorized occupation thereon. National Highways Authority of India Act of 1998 provides for the constitution of an authority for the development, maintenance, and management of NHs and for matters connected therewith or incidental thereto. Way Forward: A well-designed highway should facilitate high mobility while safely segregating the slower-moving traffic. Also merging of highways with streets should be methodical as well. A six-monthly audit of all roads must be undertaken to identify accident-prone zones and ensure the provisioning of adequate warning signages. Crash barriers, parapets, lighting provision, weatherproof roads, provision of large mirrors on sharp curves, etc. should be compulsory for all roads. Indigenization of new technologies for safety features will bring the cost down which will increase its accessibility. Implementing the Bharat NCAP (New Car Assessment Program) a rating-based safety assessment (in line with the global standards) of Indian cars. Vehicle Scrappage Policy to scrap old and unfit vehicles provides incentives to the owner of old vehicles to purchase a new vehicle. Strong public focus on wearing rear seatbelts, road safety awareness hubs, interaction with schools and colleges, etc. Stricter licensing norms, diligent driver training, and ensuring coordination among various departments and ministries should be implemented. Source: ET INSIGHTS Practice MCQs Daily Practice MCQs Q1.) With reference to the Chabahar port, consider the following statements: It is the Iranian port that is the closest to India. It is located at the mouth of Gulf of Oman. Chabahar is envisaged as a crucial link in the eastern route of the International North-South Transport Corridor (INSTC), facilitating the movement of goods between India and Central Asian countries How many of the statements given above are correct? Only one Only two All three None Q2.) 55 Cancri e recently heard in news is An exoplanet with atmosphere A malware infecting power grid A new covid variant A new type of cancer Q3.) The transplantation of an organ or tissue from one individual to another of the same species with a different genotype is referred to as: Allotransplantation Xenotransplantation Isotransplantation Autotransplantation Comment the answers to the above questions in the comment section below!! ANSWERS FOR ’  15th May 2024 – Daily Practice MCQs’ will be updated along with tomorrow’s Daily Current Affairs.st ANSWERS FOR  14th May – Daily Practice MCQs Answers- Daily Practice MCQs Q.1) – b Q.2) – d Q.3) – c

Daily Prelims CA Quiz

UPSC Quiz – 2024 : IASbaba’s Daily Current Affairs Quiz 15th May 2024

The Current Affairs questions are based on sources like ‘The Hindu’, ‘Indian Express’ and ‘PIB’, which are very important sources for UPSC Prelims Exam. The questions are focused on both the concepts and facts. The topics covered here are generally different from what is being covered under ‘Daily Current Affairs/Daily News Analysis (DNA) and Daily Static Quiz’ to avoid duplication. The questions would be published from Monday to Saturday before 2 PM. One should not spend more than 10 minutes on this initiative. Gear up and Make the Best Use of this initiative. Do remember that, “the difference between Ordinary and EXTRA-Ordinary is PRACTICE!!” Important Note: Don’t forget to post your marks in the comment section. Also, let us know if you enjoyed today’s test 🙂After completing the 5 questions, click on ‘View Questions’ to check your score, time taken, and solutions.To take the Test Click Here

DAILY CURRENT AFFAIRS IAS | UPSC Prelims and Mains Exam –14th May 2024

Archives (PRELIMS & MAINS Focus)   OLEANDER FLOWERS Syllabus Prelims – Science Context: Two Kerala government-controlled temple boards, which together manage 2,500-odd temples in the state, have banned use of oleander flowers (locally known as arali) in temple offerings after a 24-year old woman died after accidentally chewing some oleander leaves. Background:- Oleander’s toxicity has been recognised across the world. About Oleander Nerium oleander, commonly known as oleander or rosebay, is a plant cultivated worldwide in tropical, subtropical, and temperate regions. Known for its drought tolerance, the shrub is often used for ornamental and landscaping purposes. In Kerala, the plant is known by the names of arali and kanaveeram and is grown along highways and beaches as a natural, green fencing. There are different varieties of oleander, each with a flower of a different colour References in Ayurveda : The Ayurvedic Pharmacopoeia of India (API), a government document that describes the quality, purity, and strength of drugs used in Ayurveda, mentions oleander. According to API, an oil prepared from the root bark can be used to treat skin diseases. Charka [Charak Samhita] has prescribed the leaves of white-flowered variety externally in chronic and obstinate skin diseases of serious nature including leprosy. Bhavaprakasha has described Karvira [another name of the plant] as a visha (poison) and indicated it in treatment of vrana (infected wounds), kustha (skin diseases including leprosy), krimi (microbes and parasites), kandu (itching) etc Ingestion or inhalation of smoke from burning oleander can be intoxicating. This is due to the properties of cardiac glycosides (a type of chemical) including oleandrin, folinerin, and digitoxigenin, which are present in all parts of the plant. Cardiac glycosides are steroidal compounds capable of exerting pharmacological effects on cardiac muscle. The primary therapeutic value of these glycosides lies in their ability to exert profound tonic effects on the heart [stronger and faster heart contractions]. Effects of oleander toxicity include nausea, diarrhea, vomiting, rashes, confusion, dizziness, irregular heartbeat, slow heartbeat, and, in extreme cases, death. Source:Indian Express ZERO-DAY VULNERABILITY Syllabus Prelims – Current Event Context: Google released a security update for Chrome to fix a zero-day vulnerability. Background: The security vulnerability was being exploited in the wild, and could result in data leakage, code execution and crashes in the software.This is the fifth zero-day exploit Google fixed since the start of this year. Three zero-day exploits were discovered in March 2024. Key Takeaways A Zero-Day Vulnerability is an undiscovered flaw in an application or operating system. The term “Zero day” refers to the fact that the software or device vendor has zero days to fix the flaw because malicious actors can already use it to access vulnerable systems. The unknown or unaddressed vulnerability is referred to as a zero-day vulnerability or zero-day threat. A Zero-Day Exploit is a cyberattack vector that takes advantage of this unknown or unaddressed security flaw. A zero-day attack is when a malicious actor uses a zero-day exploit to plant malware, steal data, or otherwise cause damage to users, organizations, or systems. Zero-day vulnerabilities—especially in widely-used operating systems or computing devices—are a severe security risk. They leave huge numbers of users or entire organizations wide open to cybercrime until the vendor or the cybersecurity community identifies the problem and releases a solution. A similar but separate concept, Zero-Day Malware, is a virus or malware for which the signature is unknown or as yet unavailable, and therefore undetectable by many antivirus software solutions or other signature-based threat detection technologies Source: Hindu Kaziranga National Park Syllabus Prelims – Environment Context: Assam’s Kaziranga National Park collects highest revenue in its 50-year history.The revenue collected by the authorities of Kaziranga National Park, from the tourists, touches ₹8.8 crore. Background: Kaziranga National park is sprinkled with elephant-grass meadows, swampy lagoons, and dense forests is home to more than 2200 Indian one-horned rhinoceros, approximately 2/3rd of their total world population. About Kaziranga National Park: Kaziranga National Park is a national park located in the Golaghat and Nagaon districts of the state of Assam, India. Located on the edge of the Eastern Himalaya biodiversity hotspot, the park combines high species diversity and visibility. The park is a UNESCO World Heritage Site. It’s renowned for its population of the Indian one-horned rhinoceros, which is one of the main attractions for visitors. Kaziranga is also recognized as an Important Bird Area by BirdLife International for the conservation of avifaunal species. The park’s contribution in saving the Indian one-horned rhinoceros from the brink of extinction at the turn of the 20th century to harbouring the single largest population of this species is a spectacular conservation achievement. The park is spread over an area of 1,090 km² (420 sq mi) and is home to a large breeding population of elephants, wild water buffalo, and swamp deer. The Endangered Ganges dolphin is also found in some of the closed oxbow lakes. Over the time, the tiger population has also increased in Kaziranga, and that’s the reason why Kaziranga was declared as Tiger Reserve in 2006. Due to the difference in altitude between the eastern and western areas of the park, one can see mainly four types of vegetation like alluvial inundated grasslands, alluvial savanna woodlands, tropical moist mixed deciduous forests, and tropical semi-evergreen forests. Kumbhi, Indian gooseberry, the cotton tree, and elephant Apple are amongst the famous trees that can be seen in the park. It is criss-crossed by four major rivers, including the Brahmaputra. The park faces several challenges, including poaching, habitat loss due to human encroachment, and annual flooding from the Brahmaputra River. Efforts to mitigate these threats involve anti-poaching measures, community involvement, and habitat restoration projects. Source: Hindu SICKLE CELL DISEASE Syllabus Prelims – Science Context: Recently, a 12-year-old boy from USA, became the first person in the world with sickle cell disease to begin a commercially approved gene therapy that may cure the condition. Background: The U.S. Food and Drug Administration (FDA) has approved a gene therapy called Casgevy, which represents a significant milestone in the treatment of sickle cell disease. About Sickle Cell Disease : Sickle cell disease is a group of inherited disorders that affect the hemoglobin, the protein that carries oxygen through the body. People with SCD have atypical haemoglobin molecules called haemoglobin S, which can distort RBCs into a sickle or crescent shape. This distortion leads to several complications, including chronic anaemia, acute painful episodes (known as sickle cell crises), organ infarction (tissue damage due to blocked blood flow), and chronic organ damage. Unfortunately, SCD significantly reduces life expectancy for affected individuals. Treatment Options: Blood Transfusions: These helps relieve anaemia and reduce the risk of pain crises. Hydroxyurea: A medication that reduces the frequency of painful episodes and prevents long-term complications. Bone Marrow or Stem Cell Transplantation: In some cases, this approach can be used to replace the defective stem cells with healthy ones. Government Initiatives in India: The National Health Mission in India supports states in preventing and managing Sickle Cell Disease (SCD). The government aims to eliminate Sickle Cell Anaemia by 2047. Technical operational guidelines have been released for prevention and control of hemoglobinopathies, including sickle cell anaemia. Source: New York Times Goods and Service Tax Syllabus Prelims – Economy Context: The Goods and Services Tax (GST) collections as a percentage of Gross Domestic Product (GDP) have reached pre-GST levels, Finance Minister Nirmala Sitharaman said.The monthly gross GST collections have been picking pace, recently crossing a landmark of Rs 2 lakh crore, as per the monthly data for April released on May 1 by the Finance Ministry. Background: In India, GST is a successor to VAT used on the supply of goods and service. It is a comprehensive, multistage, destination-based tax. It has subsumed almost all the indirect taxes except a few state taxes. GST replaced many indirect taxes in India such as the excise duty, VAT, services tax, etc. The Goods and Service Tax Act was passed in the Parliament on 29th March 2017 and came into effect on 1st July 2017. About Goods and Service Tax: GST is one indirect tax for the whole nation, which will make India one unified common market. GST is a single tax on the supply of goods and services, right from the manufacturer to the consumer. In the earlier indirect tax regime, the Centre could tax goods up to the production or manufacturing stage, while States collected taxes on the sale or distribution of goods. The right to tax services was vested with the Centre alone. Under the GST, both the Centre and the States can tax the entire supply chain in both goods as well as services – right from production to distribution. Credits of input taxes paid at each stage will be available in the subsequent stage of value addition, which makes GST essentially a tax only on value addition at each stage. The final consumer will thus bear only the GST charged by the last dealer in the supply chain, with set-off benefits at all the previous stages. Benefits of GST For business and industry Easy compliance: A robust and comprehensive IT system would be the foundation of the GST regime in India. Therefore, all tax payer services such as registrations, returns, payments, etc. would be available to the taxpayers online, which would make compliance easy and transparent. Uniformity of tax rates and structures: GST will ensure that indirect tax rates and structures are common across the country, thereby increasing certainty and ease of doing business. In other words, GST would make doing business in the country tax neutral, irrespective of the choice of place of doing business. Removal of cascading: A system of seamless tax-credits throughout the value-chain, and across boundaries of States, would ensure that there is minimal cascading of This would reduce hidden costs of doing business. Improved competitiveness: Reduction in transaction costs of doing business would eventually lead to an improved competitiveness for the trade (exports) and industry. For Central and State Governments Simple and easy to administer: Multiple indirect taxes at the Central and State levels are being replaced by GST. Backed with a robust end-to-end IT system, GST would be simpler and easier to administer than all other indirect taxes of the Centre and State levied so far. Better controls on leakage: GST will result in better tax compliance due to a robust IT infrastructure. Due to the seamless transfer of input tax credit from one stage to another in the chain of value addition, there is an in-built mechanism in the design of GST that would incentivize tax compliance by traders. Higher revenue efficiency: GST is expected to decrease the cost of collection of tax revenues of the Government, and will therefore, lead to higher revenue efficiency. For the consumer: Single and transparent tax proportionate to the value of goods and services: Due to multiple indirect taxes being levied by the Centre and State, with incomplete or no input tax credits available at progressive stages of value addition, the cost of most goods and services in the country today are laden with many hidden taxes. Under GST, there would be only one tax from the manufacturer to the consumer, leading to transparency of taxes paid to the final consumer. Relief in overall tax burden: Because of efficiency gains and prevention of leakages, the overall tax burden on most commodities will come down, which will benefit consumers. Source: Indian Express Critical Minerals Syllabus Mains – GS 3 Context: India has reached out to key critical mineral producers to bring in processing technology into the country.The move comes close on the back of the government rolling out auctions of critical mineral mines. Background: Talks are on with the United States (US), Australia, and United Kingdom (UK), South Korea, and Japan for processing technology. Brazil and Argentina are also positive about collaborating with India. Critical Minerals: They are elements that are the building blocks of essential modern-day technologies and are at risk of supply chain disruptions. For example, Antimony, Beryllium, Bismuth, Cobalt, Copper, etc. For India, major import sources of Cobalt are China, the US, and Japan; Lithium (Chile, Russia, China); Nickel (Sweden, China), etc. These minerals are now used everywhere from making mobile phones, and computers to batteries, electric vehicles, and green technologies like solar panels and wind turbines. Based on their individual needs and strategic considerations, different countries create their own lists. Significance of Critical Minerals for India: Minerals such as lithium, graphite, cobalt, titanium, etc. are essential for the advancement of high-tech electronics, telecommunications, transport, etc. Self-sufficiency in critical minerals ensures self-reliance and addresses the vulnerability in its supply chain. Some critical minerals are important for climate-friendly technologies like electric vehicles, solar panels wind turbines, etc. Critical minerals such as lithium, nickel, and silicon are vital to manufacturing technologies and materials used in the aerospace and defence sector. Critical minerals are necessary for India to achieve its geo-economic goals, energy security, renewable energy goals, mineral security, and commitment to electric vehicles by 2030. Issues/Challenges/Concerns associated with Critical Minerals: Some critical mineral assets for the country’s manufacturing sectors (particularly green technologies) are not ready to be mined. There are also some critical minerals of which there exist no known resources within the country. Geographical concentration of extraction or processing in a few locations may lead to supply chain disruption or vulnerabilities. For example, three-fifths of rare earth minerals, essential for clean energy, electronics, and defence technologies, are mined in China, while more than fourth fifths are processed there. The supply of critical minerals is dependent on trade agreements which can be driven by domestic interests. For example, Indonesia (host to the world’s largest nickel reserves), has banned export of raw nickel ore, as they seek to develop a domestic nickel processing industry. India lacks periodic assessments for judging the criticality of the minerals needed. This affects the prices that are paid for such minerals and long-term risk mitigation plans. These minerals lack substitutes and have low end-of-life recycling rates. Initiatives taken regarding Critical Minerals: India- Australia Critical Minerals Investment Partnership will build new supply chains underpinned by critical minerals processed in Australia to help India’s plans to lower emissions from its electricity network and become a global manufacturing hub, including for electric vehicles. India has joined the Minerals Security Partnership (MSP) which is a US-led collaboration of 14 countries that aims to catalyse public and private investment in critical mineral supply chains globally. Mines and Minerals (Development and Regulation) Amendment Bill 2021 ensures transparency in the auction process of mines and enhances domestic and foreign investment in the mining sector. India, Japan, and Australia unveiled Supply Chain Resilience Initiative (SCRI) to enhance the resilience of supply chains in the Indo-Pacific Region and reliance on China. Source: Economic Times Practice MCQs Daily Practice MCQs Q1.)  Which among the following flower was recently banned from being part of temple offering due to its poisonous nature : Parijata or Indian Magnolia Nerium Oleander Safed Aak Or Crown Flower Cobra saffron or Nagavriksha Q2.)The word Zero Day Vulnerability recently seen in news is related to: An imminent nuclear attack Use of Chemical weapon Release of Biological Weapon Cybersecurity Q3.)  Consider the following statements about kaziranga national park: The park is located on the edge of the Eastern Himalaya biodiversity hotspot. Species like tigers and Ganges River dolphin are part of the parks diverse Fauna. It is recognized as an Important Bird Area by BirdLife International for conservation of avifaunal species. How many of the statements given above are correct? Only one Only two All three None Comment the answers to the above questions in the comment section below!! ANSWERS FOR ’ 14th May 2024 – Daily Practice MCQs’ will be updated along with tomorrow’s Daily Current Affairs.st ANSWERS FOR 13th May– Daily Practice MCQs Answers- Daily Practice MCQs Q.1) – a Q.2) – b Q.3) – c

Daily Prelims CA Quiz

UPSC Quiz – 2024 : IASbaba’s Daily Current Affairs Quiz 14th May 2024

The Current Affairs questions are based on sources like ‘The Hindu’, ‘Indian Express’ and ‘PIB’, which are very important sources for UPSC Prelims Exam. The questions are focused on both the concepts and facts. The topics covered here are generally different from what is being covered under ‘Daily Current Affairs/Daily News Analysis (DNA) and Daily Static Quiz’ to avoid duplication. The questions would be published from Monday to Saturday before 2 PM. One should not spend more than 10 minutes on this initiative. Gear up and Make the Best Use of this initiative. Do remember that, “the difference between Ordinary and EXTRA-Ordinary is PRACTICE!!” Important Note: Don’t forget to post your marks in the comment section. Also, let us know if you enjoyed today’s test 🙂After completing the 5 questions, click on ‘View Questions’ to check your score, time taken, and solutions.To take the Test Click Here

DAILY CURRENT AFFAIRS IAS | UPSC Prelims and Mains Exam –13th May 2024

Archives (PRELIMS & MAINS Focus)   KAWASAKI DISEASE Syllabus Prelims – Science Context: Two baby girls diagnosed with kawasaki disease , a condition causing inflammation of blood vessels, were successfully treated at Coimbatore Medical College and Hospital. Background:- The disease is named after Japanese pediatrician Tomisaku Kawasaki, who first described it in 1967. About Kawasaki disease Kawasaki disease is a rare but serious condition that primarily affects children under the age of 5. It’s also known as mucocutaneous lymph node syndrome. Kawasaki disease can lead to complications, particularly affecting the heart. If untreated for a prolonged period it may lead to heart complications such as inflammation of the coronary artery, inflammation of heart muscles, problems associated with heart valves, and weakening of the blood walls in the heart. Treatment usually involves intravenous immunoglobulin (IVIG) to reduce inflammation and fever, along with aspirin to prevent blood clots. Early diagnosis and treatment are crucial to reduce the risk of complications, especially involving the heart. It is not an infectious disease, that is, it does not spread between people The cause is not fully understood, but it’s believed to involve an abnormal immune response triggered by an infection or other environmental factors in genetically predisposed individuals. Symptoms Symptoms of Kawasaki disease include a fever greater than 102.2 degrees Fahrenheit (39 degrees Celsius) for five or more days. And the child has at least four of the following symptoms: A rash on the main part of the body or in the genital area. An enlarged lymph node in the neck. Very red eyes without a thick discharge. Red, dry, cracked lips and a red, swollen tongue. Swollen, red skin on the palms of the hands and the soles of the feet. Later the skin on fingers and toes peels. Other symptoms might include:Belly pain, Diarrhea, Fussiness, Joint pain, Vomiting. Risk factors include: Age: Children below 5 years are at high risk Gender: Boys are more likely than girls to develop Kawasaki disease Ethnicity: Japanese or Korean children are at higher risk to develop Kawasaki disease Source: Mayoclinic Auroras (Northern and Southern lights) Syllabus Prelims – Science Context: The night sky was lit up by northern lights, or aurora borealis, at Hanle village in Ladakh early Saturday morning. Northern lights were also witnessed in other parts of the world, including in the United States and the United Kingdom. Meanwhile, southern lights, or aurora australis, were spotted in countries such as New Zealand and Australia. Background: Severe geomagnetic storm has allowed rare aurora to show in Ladakh, as observed from the Hanle observatory. While rare, this is not the first sighting of Northern lights in Ladakh. Key takeaways : Auroras are essentially natural lights that appear as bright, swirling curtains in the night sky and can be seen in a range of colours, including blue, red, yellow, green, and orange. These lights primarily appear near the poles of both the northern and southern hemispheres all year round but sometimes they expand to lower latitudes. In the north, the display is called the aurora borealis; in the south, it is known as the aurora australis. Why do auroras occur? It is due to activity on the surface of the Sun. The star continuously releases a stream of charged particles, mainly electrons and protons, and magnetic fields called the solar wind. As the solar wind approaches the Earth, it is deflected by the planet’s magnetic field, which acts like a protective shield. However, some of the charged particles are trapped in the magnetic field and they travel down the magnetic field lines at the north and south poles into the upper atmosphere of the Earth. These particles then interact with different gases present there, resulting in tiny flashes that light up the night sky. When solar wind particles collide with oxygen, a green colour light is produced. Interaction with nitrogen produces shades of blue and purple. Auroras expand to midlatitudes when the solar wind is extremely strong. This happens when the activity on the Sun’s surface goes up, leading to solar flares and coronal mass ejections (CMEs), which are essentially extra bursts of energy in the solar wind. In such cases, the solar wind is so intense that it can result in a geomagnetic storm, also known as a magnetic storm — a temporary disturbance of the Earth’s magnetic field. It is during a magnetic storm that auroras can be seen in the mid-latitudes. One such geomagnetic storm was kicked off on Friday after a CME hit the Earth. That’s why auroras were visible in several parts of the world. Geomagnetic storms can also affect space-dependent operations like Global Positioning Systems (GPS), radio and satellite communications, flight operations, power grids, and space exploration programmes. Source: Indian Express Gulf of Aden Syllabus Prelims – Geography Context: European naval force detained six suspected pirates on Friday after they opened fire on an oil tanker traveling through the Gulf of Aden, likely part of a growing number of piracy attacks emanating from Somalia. Background: Once-rampant piracy off the Somali coast diminished after a peak in 2011. That year, there were 237 reported attacks in waters off Somalia. Somali piracy in the region at the time cost the world’s economy some $7 billion — with $160 million paid out in ransoms.However, concerns about new attacks have grown in recent months. In the first quarter of 2024, there have been five reported incidents off Somalia.   About Gulf of Aden The Gulf of Aden is a deepwater gulf of the Indian Ocean between Yemen to the north, the Arabian Sea to the east, Djibouti to the west, and the Guardafui Channel, Socotra and Somalia to the south. In the northwest, it connects with the Red Sea through the Bab-el-Mandeb strait, and it connects with the Arabian Sea to the east. To the west, it narrows into the Gulf of Tadjoura in Djibouti. The Aden Ridge lies along the middle of the Gulf and is causing it to widen about 15mm per year. The Gulf of Aden is approximately 900 km long and 500 km wide, covering roughly 410,000 square kilometers. It has an average depth of 500 meters and a maximum depth of 2,700 meters. The Gulf of Aden is an important shipping route, with over 20,000 vessels passing through the Gulf annually. This route is often used for the delivery of Persian Gulf oil, making the gulf an integral waterway in the world economy. This strategic waterway is of significant geopolitical importance due to its proximity to the Suez Canal and its role in global trade. However, it has also been known for piracy, particularly off the coast of Somalia. Important cities along the Gulf of Aden include Aden and Mukalla in Yemen, and Djibouti City in Djibouti. Source: Livemint Lithium Syllabus Prelims – Geography Context: Khanij Bidesh India Ltd (KABIL) hopes to acquire a lithium block in Australia this year, a top official said on Saturday. The government had earlier said that India would pump in about Rs 200 crore to explore and develop five lithium mines in Argentina. Argentina is part of the ‘lithium triangle’ along with Chile and Bolivia with more than half of the world’s total lithium resources. Background: KABIL is owned by three public sector undertakings — National Aluminium Company Ltd (Nalco), Hindustan Copper Ltd (HCL) and Mineral Exploration and Consultancy Ltd (MECL). About Lithium : The Lithium is a chemical element with the symbol Li and atomic number 3. It is a soft, silvery-white alkali metal that is highly reactive and flammable. Lithium is the most important mineral for the energy transition, a fundamental component of lithium-ion batteries, which power electric vehicles and battery energy storage systems. India is 100 per cent reliant on imports for its lithium. Physical Properties: Lithium is a silvery-white metal. It has a standard atomic weight of approximately 6.94. Lithium is the lightest metal and the lightest solid element, and it is known for its low density and high electrochemical potential. At standard temperature and pressure, lithium is a solid. It has a melting point of 180.50 °C (356.90 °F) and a boiling point of 1330 °C (2426 °F). Chemical Properties: Lithium is part of the alkali metal group and has a single valence electron in its outer shell, which it readily donates to form cations and ionic bonds. Occurrence and Production: Lithium occurs in a number of pegmatitic minerals, but is also commonly obtained from brines and clays. It’s extracted commercially from spodumene and lepidolite. Lithium reserves are primarily found in salt flats (salars), particularly in countries like Chile, Argentina, Australia, and China. Due to the growing demand for lithium-ion batteries, there is increasing interest in lithium mining and production, as well as efforts to develop more efficient recycling processes to reduce reliance on newly extracted lithium Uses: Lithium and its compounds have several industrial applications, including heat-resistant glass and ceramics, lithium grease lubricants, flux additives for iron, steel and aluminium production, lithium batteries, and lithium-ion batteries. Lithium is also used in the treatment of bipolar disorder. Safety Precautions: Lithium is corrosive and requires special handling to avoid skin contact. Ingesting lithium can be fatal. Lithium toxicity, which occurs at only slightly higher doses than the medicinal levels, can cause drowsiness, slurred speech, and even seizures. Source: Indian Express Flash floods Syllabus Prelims & Mains – Geography Context: Flash floods, that have ripped through northern Afghanistan, have left more than 300 people dead, the United Nations World Food Programme (WFP) said. Background: A climate-sensitive nation, Afghanistan was battered by a similar calamity in April as heavy rains and flash floods resulted in the deaths of at least 100 people across 10 provinces in Afghanistan.Afghanistan’s vulnerability to climate change is exacerbated by its relatively dry winter, making it challenging for the soil to absorb rainfall. About Flash floods Flash floods are rapid floods that occur in low-lying areas. They can be caused by heavy rain associated with a severe thunderstorm, hurricane, or tropical storm, or by meltwater from ice or snow flowing over ice sheets or snowfields. Flash floods can also occur after the collapse of a natural ice or debris dam, or a human structure such as a man-made dam. Flash floods are distinguished from regular floods by having a timescale of fewer than six hours between rainfall and the onset of flooding. The unpredictability and short time frame of flash floods make them more destructive than regular floods Flash floods can also deposit large quantities of sediments on floodplains and can be destructive of vegetation cover not adapted to frequent flood conditions In India, flash floods are often associated with cloudbursts – sudden, intense rainfall in a short period of time. Himalayan states further face the challenge of overflowing glacial lakes, formed due to the melting of glaciers, and their numbers have been increasing in the last few years. India is the worst flood-affected country in the world after Bangladesh and accounts for one-fifth of the global death count due to floods. Flash floods have been commonly witnessed in cities like Chennai and Mumbai. Source: Indian Express Illegal Immigration Syllabus Mains – GS 3 Context: Manipur Chief Minister N Biren Singh said on Sunday that 5,801 illegal immigrants from Myanmar had been detected in Kamjong district since a civil war broke out in the neighbouring country. This has brought back the topic of illegal migration again in news Background: The topic is very sensitive and humanitarian dimension also needs to be considered along with others. Illegal Immigration is a threat to Internal Security: It is alleged that among the illegal migrants there are also militants especially groups such as NSCN/KUFLA, who enter India to carry out terrorist activities. In recent decades, trafficking of women and human smuggling has become quite rampant across borders. The Commission on Integration and Cohesion found that tension usually exists with the presence of high levels of migration combine with other forms of social exclusion like poverty, poor housing, etc. Immigration has increased pressure on the part of the government, as the government has to increase the expenditure on education and health facilities for the immigrants. Most illegal immigrants have got their names enlisted in the voting list illegally, thereby claiming themselves as citizens of the state. The influx of immigrants created a crisis of identity among the indigenous people of the region. Large areas of forestland were encroached upon by the immigrants for settlement and cultivation. This creates conflict in resource utilization with the local communities. Laws in India to tackle Illegal Migration: Under the Foreigners Act of 1946, the central government can deport illegal foreign nationals. Under the Passport (Entry into India) Act of 1920, the state governments have been entrusted to remove an illegal foreigner by force. The Citizenship Act of 1955 provides for the acquisition and determination of Indian citizenship. Way Forward: A considerable portion of the problem of illegal immigration lies in the fact that we have no such effective refugee policy. The central government should frame a holistic refugee policy. India has to make a diplomatic effort to get neighboring countries to cooperate as illegal migration cannot be solved unless the origin country cooperates. Government should introduce identity cards for the borderland people who frequently used to cross borders for different reasons. There is a vast area of the border that is not protected by fencing. The central government should complete the border fencing. Regional forums like BIMSTEC can be used to discuss issues like illegal migration from neighboring countries and garner support and coordination from the members. Border guards like BSF, and ITBP should be strengthened and a second line of defence can be created by the State government through the state police force. Source: The Hindu Practice MCQs Daily Practice MCQs Q1.) Consider the following statements about Kawasaki disease: It is an infectious disease primarily affecting lactating mothers. It can lead to complications particularly affecting heart. It is caused by radiation exposure. How many of the statements given above are correct? Only one Only two All three None Q2.)With reference to aurora borealis and aurora australis, consider the following statements: Auroras expand to midlatitudes when the solar wind is extremely weak. Aurora borealis is the other name for northern lights. Ladakh has witnessed Auroras multiple times. How many of the statements given above are correct? Only one Only two All three None Q3.) Consider the following statements about Lithium Spodumene and lepidolite are the main mineral resources for extracting lithium. It is a soft, silvery-white alkali metal that is highly reactive and flammable. Lithium is used in the treatment of bipolar disorder. How many of the statements given above are correct? Only one Only two All three None Comment the answers to the above questions in the comment section below!! ANSWERS FOR ’  13th May 2024 – Daily Practice MCQs’ will be updated along with tomorrow’s Daily Current Affairs.st ANSWERS FOR  11th May – Daily Practice MCQs Answers- Daily Practice MCQs Q.1) – d Q.2) – c Q.3) – c

Daily Prelims CA Quiz

UPSC Quiz – 2024 : IASbaba’s Daily Current Affairs Quiz 13th May 2024

The Current Affairs questions are based on sources like ‘The Hindu’, ‘Indian Express’ and ‘PIB’, which are very important sources for UPSC Prelims Exam. The questions are focused on both the concepts and facts. The topics covered here are generally different from what is being covered under ‘Daily Current Affairs/Daily News Analysis (DNA) and Daily Static Quiz’ to avoid duplication. The questions would be published from Monday to Saturday before 2 PM. One should not spend more than 10 minutes on this initiative. Gear up and Make the Best Use of this initiative. Do remember that, “the difference between Ordinary and EXTRA-Ordinary is PRACTICE!!” Important Note: Don’t forget to post your marks in the comment section. Also, let us know if you enjoyed today’s test 🙂After completing the 5 questions, click on ‘View Questions’ to check your score, time taken, and solutions.To take the Test Click Here

[DAY 60] 60 DAY RAPID REVISION (RaRe) SERIES for UPSC Prelims 2024 – ENVIRONMENT, CURRENT AFFAIRS & CSAT TEST SERIES!

Archives Hello Friends The 60 Days Rapid Revision (RaRe) Series is IASbaba’s Flagship Initiative recommended by Toppers and loved by the aspirants’ community every year. It is the most comprehensive program which will help you complete the syllabus, revise and practice tests on a daily basis. The Programme on a daily basis includes Daily Prelims MCQs from Static (Monday – Saturday) Daily Static Quiz will cover all the topics of static subjects – Polity, History, Geography, Economics, Environment and Science and technology. 20 questions will be posted daily and these questions are framed from the topics mentioned in the schedule. It will ensure timely and streamlined revision of your static subjects. Daily Current Affairs MCQs (Monday – Saturday) Daily 5 Current Affairs questions, based on sources like ‘The Hindu’, ‘Indian Express’ and ‘PIB’, would be published from Monday to Saturday according to the schedule. Daily CSAT Quiz (Monday – Friday) CSAT has been an Achilles heel for many aspirants. Daily 5 CSAT Questions will be published. Note – Daily Test of 20 static questions, 10 current affairs, and 5 CSAT questions. (35 Prelims Questions) in QUIZ FORMAT will be updated on a daily basis. To Know More about 60 Days Rapid Revision (RaRe) Series – CLICK HERE   60 Day Rapid Revision (RaRe) Series Schedule – CLICK HERE  Important Note Comment your Scores in the Comment Section. This will keep you accountable, responsible and sincere in days to come. It will help us come out with the Cut-Off on a Daily Basis. Let us know if you enjoyed today’s test 🙂  You can post your comments in the given format  (1) Your Score (2) Matrix Meter (3) New Learning from the Test Time limit: 0 Test-summary 0 of 35 questions completed Questions: 1 2 3 4 5 6 7 8 9 10 11 12 13 14 15 16 17 18 19 20 21 22 23 24 25 26 27 28 29 30 31 32 33 34 35 Information The following Test is based on the syllabus of 60 Days Plan-2023 for UPSC IAS Prelims 2022. To view Solutions, follow these instructions: Click on – ‘Start Test’ button Solve Questions Click on ‘Test Summary’ button Click on ‘Finish Test’ button Now click on ‘View Questions’ button – here you will see solutions and links. You have already completed the test before. Hence you can not start it again. Test is loading... You must sign in or sign up to start the test. You have to finish following test, to start this test: Results 0 of 35 questions answered correctly Your time: Time has elapsed You have scored 0 points out of 0 points, (0) Average score     Your score     Categories Not categorized 0% Your result has been entered into leaderboard Loading Name: E-Mail: Captcha: maximum of 70 points Pos. Name Entered on Points Result Table is loading No data available 1 2 3 4 5 6 7 8 9 10 11 12 13 14 15 16 17 18 19 20 21 22 23 24 25 26 27 28 29 30 31 32 33 34 35 Answered Review Question 1 of 35 1. Question Consider following pairs: Species Name                        IUCN status Himalayan Wolf          Least Concerned Polar bear                      Endangered Lesser Florican              Endangered How many of the above are correct? a) Only one b) Only two c) All three d) None Correct Solution (d) The Himalayan Wolf (Canis lupus chanco), a prominent lupine predator found across the Himalayas the taxonomic status of which was a puzzle till late, has been assessed for the first time in the International Union for Conservation of Nature (IUCN)’s Red List. The animal has been categorised as ‘Vulnerable’ on the IUCN Red List. (Hence pair 1 is incorrect) Polar bears endemic to the region are listed as “vulnerable” in the International Union for Conservation of Nature (IUCN) Red List of Endangered Species for its loss of habitat — sea ice. (Hence pair 2 is incorrect) Lesser floricans are one of the four species of birds of the bustard family that occur in India. They have been categorised as critically endangered species on the Red List of Threatened Species drawn by the International Union for Conservation of Nature (IUCN). (Hence pair 3 is incorrect) Incorrect Solution (d) The Himalayan Wolf (Canis lupus chanco), a prominent lupine predator found across the Himalayas the taxonomic status of which was a puzzle till late, has been assessed for the first time in the International Union for Conservation of Nature (IUCN)’s Red List. The animal has been categorised as ‘Vulnerable’ on the IUCN Red List. (Hence pair 1 is incorrect) Polar bears endemic to the region are listed as “vulnerable” in the International Union for Conservation of Nature (IUCN) Red List of Endangered Species for its loss of habitat — sea ice. (Hence pair 2 is incorrect) Lesser floricans are one of the four species of birds of the bustard family that occur in India. They have been categorised as critically endangered species on the Red List of Threatened Species drawn by the International Union for Conservation of Nature (IUCN). (Hence pair 3 is incorrect) Question 2 of 35 2. Question Which of the following best describes “Piscivores raptors”? a) Predatory birds that feed primarily on small mammals. b) Birds of prey specialized in hunting and consuming fish. c) Raptors that exclusively target reptiles as their main food source. d) Avian hunters with a preference for insects and arachnids. Correct Solution (b) Piscivores are organisms that primarily feed on fish. In the context of raptors, which are birds of prey, piscivores are those that specialize in hunting and consuming fish. (Hence option b is correct) These raptors often have adaptations such as sharp talons and keen eyesight, enabling them to catch fish from water bodies. Examples of piscivorous raptors include ospreys and certain eagles. Incorrect Solution (b) Piscivores are organisms that primarily feed on fish. In the context of raptors, which are birds of prey, piscivores are those that specialize in hunting and consuming fish. (Hence option b is correct) These raptors often have adaptations such as sharp talons and keen eyesight, enabling them to catch fish from water bodies. Examples of piscivorous raptors include ospreys and certain eagles. Question 3 of 35 3. Question Consider the following statements about Sundarban Tigers: It is different subspecies of Panthera tigris tigris. Sundarban tigers exhibit trait of man eater tigers more than other tigers. They have adapted to a semi-aquatic lifestyle and are capable of hunting in water. How many of the above statements are correct? a) Only one b) Only two c) All three d) None Correct Solution (a) There are 5 subspecies of Tigers (current living population) that have been identified and they are as follows Indian or Royal Bengal Tiger (Panthera tigris tigris), Indochinese Tiger (Panthera tigris corbetti), Siberian (Amur) Tiger (Panthera tigris altaica), South China Tiger (Panthera tigris amoyensis), Sumatran Tiger (Panthera tigris sumatrae).(Hence statement 1 is incorrect) Considering the mutation rates that led to a genetic change, usually an animal that was isolated for a period of 1 million years was classified as different species and one that was genetically isolated for between 20,000 and 50,000 years was a different sub-species. In the case of Sundarban Tiger, it was a part of a contiguous region with others and was perhaps separated about 500 to 1000 years ago. An estimate provided by the forest department claims that 410 people were attacked by tigers between 1985 and 2010, leaving just 95 survivors. It is not established that Sunderban tigers are distinctly more man-eaters. Statistics make it myth. (Hence statement 2 is incorrect) The Sundarban Tiger exhibits certain distinctive morphological adaptations that make it particularly suited to the mangrove habitat of Sundarban islands and capable of hunting in water. (Hence statement 3 is correct) Incorrect Solution (a) There are 5 subspecies of Tigers (current living population) that have been identified and they are as follows Indian or Royal Bengal Tiger (Panthera tigris tigris), Indochinese Tiger (Panthera tigris corbetti), Siberian (Amur) Tiger (Panthera tigris altaica), South China Tiger (Panthera tigris amoyensis), Sumatran Tiger (Panthera tigris sumatrae).(Hence statement 1 is incorrect) Considering the mutation rates that led to a genetic change, usually an animal that was isolated for a period of 1 million years was classified as different species and one that was genetically isolated for between 20,000 and 50,000 years was a different sub-species. In the case of Sundarban Tiger, it was a part of a contiguous region with others and was perhaps separated about 500 to 1000 years ago. An estimate provided by the forest department claims that 410 people were attacked by tigers between 1985 and 2010, leaving just 95 survivors. It is not established that Sunderban tigers are distinctly more man-eaters. Statistics make it myth. (Hence statement 2 is incorrect) The Sundarban Tiger exhibits certain distinctive morphological adaptations that make it particularly suited to the mangrove habitat of Sundarban islands and capable of hunting in water. (Hence statement 3 is correct) Question 4 of 35 4. Question Consider the following statements about ‘Crystalline Silicon technology’: It primarily utilizes organic materials for energy conversion. It is limited to large-scale applications due to its efficiency. It is exclusively employed in wind energy systems. How many of the above statements are correct? a) Only one b) Only two c) All three d) None Correct Solution (d) Crystalline Silicon technology primarily uses crystalline semiconductor materials, not organic polymers. (Hence statement 1 is incorrect) It is not limited to small-scale applications; it is widely used in both small-scale and large-scale solar energy systems. (Hence statement 2 is incorrect) Crystalline Silicon solar cells are made from semiconductor materials with a crystalline structure. There are two main types: monocrystalline silicon and polycrystalline silicon. It is used in both photovoltaic (PV) systems and concentrated solar power systems. (Hence statement 3 is incorrect) Incorrect Solution (d) Crystalline Silicon technology primarily uses crystalline semiconductor materials, not organic polymers. (Hence statement 1 is incorrect) It is not limited to small-scale applications; it is widely used in both small-scale and large-scale solar energy systems. (Hence statement 2 is incorrect) Crystalline Silicon solar cells are made from semiconductor materials with a crystalline structure. There are two main types: monocrystalline silicon and polycrystalline silicon. It is used in both photovoltaic (PV) systems and concentrated solar power systems. (Hence statement 3 is incorrect) Question 5 of 35 5. Question Consider the following statements about Corals: Corals can reproduce either asexually or sexually. Black corals are carnivores. Reef building corals can be found in both shallow and deep waters across the oceans. How many of the above statements are correct? a) Only one b) Only two c) All three d) None Correct Solution (b) Corals have multiple reproductive strategies – they can be male or female or both, and can reproduce either asexually or sexually. Asexual reproduction is important for increasing the size of the colony, and sexual reproduction increases genetic diversity and starts new colonies that can be far from the parents.(Hence statement 1 is correct) Black corals are carnivores. Black corals can be found growing both in shallow waters and down to depths of over 26,000 feet (8,000 meters), and some individual corals can live for over 4,000 years. (Hence statement 2 is correct) Black corals are filter feeders and eat tiny zooplankton that are abundant in deep waters. While colourful shallow-water corals rely on the sun and photosynthesis for energy. Corals are found across the world’s ocean, in both shallow and deep water, but reef-building corals are only found in shallow tropical and subtropical waters. (Hence statement 3 is incorrect) This is because the algae found in their tissues need light for photosynthesis and they prefer water temperatures between 70-85°F (22-29°C). Incorrect Solution (b) Corals have multiple reproductive strategies – they can be male or female or both, and can reproduce either asexually or sexually. Asexual reproduction is important for increasing the size of the colony, and sexual reproduction increases genetic diversity and starts new colonies that can be far from the parents.(Hence statement 1 is correct) Black corals are carnivores. Black corals can be found growing both in shallow waters and down to depths of over 26,000 feet (8,000 meters), and some individual corals can live for over 4,000 years. (Hence statement 2 is correct) Black corals are filter feeders and eat tiny zooplankton that are abundant in deep waters. While colourful shallow-water corals rely on the sun and photosynthesis for energy. Corals are found across the world’s ocean, in both shallow and deep water, but reef-building corals are only found in shallow tropical and subtropical waters. (Hence statement 3 is incorrect) This is because the algae found in their tissues need light for photosynthesis and they prefer water temperatures between 70-85°F (22-29°C). Question 6 of 35 6. Question Consider the following statements draughts in India: India recognises and monitors only four types of droughts meteorological, hydrological, agricultural and ecological. Ecological drought is episodic deficit in water availability that drives ecosystems beyond thresholds of vulnerability, impacts ecosystem services. Choose the correct code: a) 1 only b) 2 only c) Both 1 and 2 d) Neither 1 nor 2 Correct Solution (b) India monitors three types of droughts – meteorological, hydrological and agricultural. (Hence statement 1 is incorrect) Ecological drought defines it as “an episodic deficit in water availability that drives ecosystems beyond thresholds of vulnerability, impacts ecosystem services and triggers feedbacks in natural and/or human systems.” (Hence statement 2 is correct) What sets ecological drought apart is its emphasis on the interconnectedness between humans and ecosystems within the context of drought Incorrect Solution (b) India monitors three types of droughts – meteorological, hydrological and agricultural. (Hence statement 1 is incorrect) Ecological drought defines it as “an episodic deficit in water availability that drives ecosystems beyond thresholds of vulnerability, impacts ecosystem services and triggers feedbacks in natural and/or human systems.” (Hence statement 2 is correct) What sets ecological drought apart is its emphasis on the interconnectedness between humans and ecosystems within the context of drought Question 7 of 35 7. Question Consider the following statements about Bears in India: Sloth bear species is the smallest bear species found across the globe. Himalayan brown Bear is least concerned based on its global stable population. Recently a Tibetan brown bear has been spotted in India for the first time. Sun Bear occurs in the North-eastern region is adapted to ‘myrmecophagy’. How many of the above statements are correct? a) Only one b) Only two c) Only three d) All four Correct Solution (b) The Sun Bear (Helarctos malayanus) is mostly distributed in South-East Asia. In India it occurs in the North-eastern region, though it is not common. It is the smallest of the eight bear species found across the globe. The Sun Bear is threatened by trade both for pets and to provide various products. IUCN lists the Sun Bear as Vulnerable. (Hence statement 1 is incorrect) The Brown Bear (Ursus arctos) is distinguished from the Himalayan Black Bear by its heavier built and brown coat. IUCN lists this species as one of Least Concern based on its globally stable population trend. However in much of its range in Asia (and India) it occurs patchily with little information about its population or connectivity of these scattered pockets. It is speculated that it is already extinct in Bhutan. (Hence statement 2 is correct) India has recorded its first-ever confirmed record of a rare Tibetan Brown Bear. The bear was spotted in high altitudes of Northern Sikkim in the camera traps installed by the Sikkim Forest Department and WWF-India. (Hence statement 3 is correct) The Sloth Bear (Melursus ursinus) is endemic to the Indian subcontinent and occurs in India, Nepal, Bhutan and Sri Lanka. This particular species is the inspiration for the much loved character Baloo in Rudyard Kipling’s Junglebook. The Sloth Bear is unique among the bear species in being adapted to ‘myrmecophagy’, i.e to eating ants and termites, which form a large proportion of its diet. IUCN lists the Sloth Bear as Vulnerable. (Hence statement 4 is incorrect) Incorrect Solution (b) The Sun Bear (Helarctos malayanus) is mostly distributed in South-East Asia. In India it occurs in the North-eastern region, though it is not common. It is the smallest of the eight bear species found across the globe. The Sun Bear is threatened by trade both for pets and to provide various products. IUCN lists the Sun Bear as Vulnerable. (Hence statement 1 is incorrect) The Brown Bear (Ursus arctos) is distinguished from the Himalayan Black Bear by its heavier built and brown coat. IUCN lists this species as one of Least Concern based on its globally stable population trend. However in much of its range in Asia (and India) it occurs patchily with little information about its population or connectivity of these scattered pockets. It is speculated that it is already extinct in Bhutan. (Hence statement 2 is correct) India has recorded its first-ever confirmed record of a rare Tibetan Brown Bear. The bear was spotted in high altitudes of Northern Sikkim in the camera traps installed by the Sikkim Forest Department and WWF-India. (Hence statement 3 is correct) The Sloth Bear (Melursus ursinus) is endemic to the Indian subcontinent and occurs in India, Nepal, Bhutan and Sri Lanka. This particular species is the inspiration for the much loved character Baloo in Rudyard Kipling’s Junglebook. The Sloth Bear is unique among the bear species in being adapted to ‘myrmecophagy’, i.e to eating ants and termites, which form a large proportion of its diet. IUCN lists the Sloth Bear as Vulnerable. (Hence statement 4 is incorrect) Question 8 of 35 8. Question What is the meaning of the term ‘Dark Fermentation’? a) It is a process aimed at decreasing atmospheric CO2 levels. b) It is a technique for generating Hydrogen fuel from wastewater. c) It is a method for the disposal of nuclear wastes. d) It is a process for producing methane from organic waste. Correct Solution (b) Dark fermentation is the fermentative conversion of organic substrate to bio-hydrogen. It is a complex process manifested by diverse groups of bacteria, involving a series of biochemical reactions using three steps similar to anaerobic conversion. Wastewater is used as a potential substrate for bio-hydrogen production in the dark fermentation process. Incorrect Solution (b) Dark fermentation is the fermentative conversion of organic substrate to bio-hydrogen. It is a complex process manifested by diverse groups of bacteria, involving a series of biochemical reactions using three steps similar to anaerobic conversion. Wastewater is used as a potential substrate for bio-hydrogen production in the dark fermentation process. Question 9 of 35 9. Question Arrange following National Park in North to South order: Jim Corbett National Park Dudhwa National Park Govind Pashu Vihar National Park Rajaji National Park Choose correct code from below. a) 1, 2, 3, 4 b) 2, 3. 4. 1 c) 3, 4, 1, 2 d) 4, 1, 2, 3 Correct Solution (c) Three national parks are of Uttarakhand and only one is from Uttar Pradesh. Knowing that national park of UP gives correct answer. As Dudhwa National Park is located in Uttar Pradesh and other three are north of it, option c is correct. Important Points/Value Additions: Govind Pashu Vihar National Park is located in Uttarkashi district, Uttarakhand. Nestled in the western Himalayas is known for its diverse flora and fauna. It encompasses alpine meadows, glaciers, and high-altitude landscapes, providing habitat to species like snow leopards, Himalayan tahr, and numerous bird species. Rajaji National Park spread across the Shivalik ranges in Uttarakhand and named after C. Rajagopalachari, the last Governor-General of India. It is a national park and tiger reserve, known for its rich biodiversity, including elephants, tigers, leopards, and several species of deer. The park is characterized by diverse landscapes, including river valleys and dense forests. Jim Corbett National Park located in the Nainital district of Uttarakhand. It is India’s oldest national park, established in 1936, renowned for its diverse wildlife, including Bengal tigers, elephants, and various bird species. It is spread across the Himalayan foothills, encompassing riverine belts, grasslands, and dense forests. Dudhwa National Park situated on the India-Nepal border in the Lakhimpur Kheri district of Uttar Pradesh, India. It is a protected area known for its swamp deer population. Dudhwa National Park features grasslands, dense forests, and the enchanting Terai ecosystem. The park is home to tigers, leopards, and a variety of bird species. Incorrect Solution (c) Three national parks are of Uttarakhand and only one is from Uttar Pradesh. Knowing that national park of UP gives correct answer. As Dudhwa National Park is located in Uttar Pradesh and other three are north of it, option c is correct. Important Points/Value Additions: Govind Pashu Vihar National Park is located in Uttarkashi district, Uttarakhand. Nestled in the western Himalayas is known for its diverse flora and fauna. It encompasses alpine meadows, glaciers, and high-altitude landscapes, providing habitat to species like snow leopards, Himalayan tahr, and numerous bird species. Rajaji National Park spread across the Shivalik ranges in Uttarakhand and named after C. Rajagopalachari, the last Governor-General of India. It is a national park and tiger reserve, known for its rich biodiversity, including elephants, tigers, leopards, and several species of deer. The park is characterized by diverse landscapes, including river valleys and dense forests. Jim Corbett National Park located in the Nainital district of Uttarakhand. It is India’s oldest national park, established in 1936, renowned for its diverse wildlife, including Bengal tigers, elephants, and various bird species. It is spread across the Himalayan foothills, encompassing riverine belts, grasslands, and dense forests. Dudhwa National Park situated on the India-Nepal border in the Lakhimpur Kheri district of Uttar Pradesh, India. It is a protected area known for its swamp deer population. Dudhwa National Park features grasslands, dense forests, and the enchanting Terai ecosystem. The park is home to tigers, leopards, and a variety of bird species. Question 10 of 35 10. Question “It is one of the few and one of the smallest national parks located within a city. One of the few remaining remnants of Southern Tropical Dry Evergreen Forests forms a pattern of mosaic with grasslands and scrubs. Well-developed Banyan trees typically dominate the woodlands. Blackbucks, Spotted Deer, Jackals, Palm Civets are well-adapted to open grasslands of diverse sizes.” Which of the following National Park is associated with above description? a) Sanjay Gandhi National Park b) Guindy National Park c) Mount Harriet National Park d) Bannerghatta National Park Correct Solution (b) Sanjay Gandhi National Park located in the northern part of Mumbai, this national park is known for its biodiversity and the ancient Kanheri Caves. The most dominating habitat-type here is the Forest habitat. Of flowering plants alone, an estimated 1300 species exist. Much of the forest here conforms to the southern mixed-deciduous forest, dominated by the Tectona, Albizzia, Terminalia, Holarrhena, Firmiana, Dalbergia etc. In fauna Spotted Deer, Sambhar, Barking Deer, Black-naped Hare and Leopard dominate. (Hence option a is incorrect) Guindy National Park is situated in the city of Chennai. It is one of the smallest national parks in India. It is home to various species of flora and fauna. It is one of the few national parks located within a city. One of the few remaining remnants of Southern Tropical Dry Evergreen Forests forms a pattern of mosaic with grasslands and scrubs in the National Park, but well-developed Banyan trees typically dominate the woodlands. Blackbucks and Spotted Deer are well-adapted to open grasslands of diverse sizes. (Hence option b is correct) Mount Harriet National Park is situated on the northern part of South Andaman Island, approximately 15 kilometers by road from Port Blair, the capital city of the Andaman and Nicobar Islands. The national park is known for its diverse flora and fauna. The forested area is home to a variety of plant species, including orchids, ferns, and other tropical vegetation. The park is also inhabited by various bird species, making it a destination for birdwatching. (Hence option c is incorrect) Bannerghatta National Park is a national park in India, located near Bangalore, Karnataka. The park has evergreen primary forests, and at Chiriyatapu the forest type is mixed deciduous, a combination of primary and secondary forests. The three types of forests are categorized as tropical evergreen, hilltop tropical evergreen and littoral. Overall 134 plant and tree species are reported, including 74 native and 51 introduced species. (Hence option d is incorrect) Incorrect Solution (b) Sanjay Gandhi National Park located in the northern part of Mumbai, this national park is known for its biodiversity and the ancient Kanheri Caves. The most dominating habitat-type here is the Forest habitat. Of flowering plants alone, an estimated 1300 species exist. Much of the forest here conforms to the southern mixed-deciduous forest, dominated by the Tectona, Albizzia, Terminalia, Holarrhena, Firmiana, Dalbergia etc. In fauna Spotted Deer, Sambhar, Barking Deer, Black-naped Hare and Leopard dominate. (Hence option a is incorrect) Guindy National Park is situated in the city of Chennai. It is one of the smallest national parks in India. It is home to various species of flora and fauna. It is one of the few national parks located within a city. One of the few remaining remnants of Southern Tropical Dry Evergreen Forests forms a pattern of mosaic with grasslands and scrubs in the National Park, but well-developed Banyan trees typically dominate the woodlands. Blackbucks and Spotted Deer are well-adapted to open grasslands of diverse sizes. (Hence option b is correct) Mount Harriet National Park is situated on the northern part of South Andaman Island, approximately 15 kilometers by road from Port Blair, the capital city of the Andaman and Nicobar Islands. The national park is known for its diverse flora and fauna. The forested area is home to a variety of plant species, including orchids, ferns, and other tropical vegetation. The park is also inhabited by various bird species, making it a destination for birdwatching. (Hence option c is incorrect) Bannerghatta National Park is a national park in India, located near Bangalore, Karnataka. The park has evergreen primary forests, and at Chiriyatapu the forest type is mixed deciduous, a combination of primary and secondary forests. The three types of forests are categorized as tropical evergreen, hilltop tropical evergreen and littoral. Overall 134 plant and tree species are reported, including 74 native and 51 introduced species. (Hence option d is incorrect) Question 11 of 35 11. Question Consider the following statements in context of ‘Dark Sky Park’: Pench Tiger Reserve becomes India’s first Dark Sky Park. It is primarily driven by the need to shield the night sky from light pollution. It is funded by the District Planning Community (DPC), designates the area for immersive stargazing. How many of the above statements are correct? a) Only one b) Only two c) All three d) None Correct Solution (c) The Pench Tiger Reserve (PTR) in Maharashtra has attained the distinction of becoming India’s inaugural Dark Sky Park and the fifth in Asia. (Hence statement 1 is correct) The decision to designate Pench as the first Dark Sky Park is driven by the need to shield the night sky from light pollution. (Hence statement 2 is correct) As India’s first and Asia’s eighth Dark Sky Park, the Pench Tiger Reserve underscores its commitment to preserving an unspoiled night sky. The recently inaugurated night sky observatory, funded by the District Planning Community (DPC), designates the area near Bagholi for immersive stargazing. (Hence statement 3 is correct) A dark-sky preserve is an area, typically surrounding a park or observatory, that limits artificial light pollution. Incorrect Solution (c) The Pench Tiger Reserve (PTR) in Maharashtra has attained the distinction of becoming India’s inaugural Dark Sky Park and the fifth in Asia. (Hence statement 1 is correct) The decision to designate Pench as the first Dark Sky Park is driven by the need to shield the night sky from light pollution. (Hence statement 2 is correct) As India’s first and Asia’s eighth Dark Sky Park, the Pench Tiger Reserve underscores its commitment to preserving an unspoiled night sky. The recently inaugurated night sky observatory, funded by the District Planning Community (DPC), designates the area near Bagholi for immersive stargazing. (Hence statement 3 is correct) A dark-sky preserve is an area, typically surrounding a park or observatory, that limits artificial light pollution. Question 12 of 35 12. Question Consider following statements: Balaram Ambaji and Jessore (Banaskantha district), Jambughoda (Panchmahal district), Ratanmahal (Dahod district) are wildlife sanctuaries located in a state. They offer safe homes to flagship species of sloth bears along with leopards, striped hyenas, jackals, wolves, jungle cats, blue bulls, wild boars, Indian foxes etc. CAG report highlighted widespread violations, poor management and encroachments. Which of the following state is associated with above mentioned description? a) Gujarat b) Madhya Pradesh c) Uttar Pradesh d) Rajasthan Correct Solution (a) A series of violations and other discrepancies have been threatening wildlife in Gujarat, especially in six sanctuaries managed by the state forest department, flagged the Comptroller Auditor General of India (CAG). Six sanctuaries in Gujarat — Balaram Ambaji and Jessore (Banaskantha district); Jambughoda (Panchmahal district), Ratanmahal (Dahod district), Shoolpaneshwar (Narmada district) and Purna (Tapi and Dangs districts) — offer safe homes to sloth bears, leopards, striped hyenas, jackals, wolves, jungle cats, blue bulls, wild boars, Indian foxes, Indian hares, common langurs, reptiles, birds, barking deers, antelopes and chitals, among others. The Ratanmahal sanctuary hosts sloth bears, a flagship species in the area. Poor spending of funds has led to mismanagement, leaving the forests open for encroachment, mining and deforestation, added CAG in its report tabled in the state assembly on September 16, 2023. Less than 1 per cent of the allotted fund was spent by the forest department in 2016-17. Though the state government allocated Rs 85,557.78 crore, just Rs 1,000 crore or 0.78 per cent was spent, CAG pointed out. Incorrect Solution (a) A series of violations and other discrepancies have been threatening wildlife in Gujarat, especially in six sanctuaries managed by the state forest department, flagged the Comptroller Auditor General of India (CAG). Six sanctuaries in Gujarat — Balaram Ambaji and Jessore (Banaskantha district); Jambughoda (Panchmahal district), Ratanmahal (Dahod district), Shoolpaneshwar (Narmada district) and Purna (Tapi and Dangs districts) — offer safe homes to sloth bears, leopards, striped hyenas, jackals, wolves, jungle cats, blue bulls, wild boars, Indian foxes, Indian hares, common langurs, reptiles, birds, barking deers, antelopes and chitals, among others. The Ratanmahal sanctuary hosts sloth bears, a flagship species in the area. Poor spending of funds has led to mismanagement, leaving the forests open for encroachment, mining and deforestation, added CAG in its report tabled in the state assembly on September 16, 2023. Less than 1 per cent of the allotted fund was spent by the forest department in 2016-17. Though the state government allocated Rs 85,557.78 crore, just Rs 1,000 crore or 0.78 per cent was spent, CAG pointed out. Question 13 of 35 13. Question “The National Park encompasses the catchments of Kiar, Nath and Kibar Nalas, all of which drain south-west into Marwah River (also known as Marusudar River). The terrain is generally rugged and steep with narrow valleys bounded by high ridges opening out in their upper glacial parts. It encompasses breathtaking alpine meadow.” The above description reflects which one of the following National Park? a) Kishtwar National Park b) Dachigam National Park c) Hemis National Park d) Salim Ali National Park Correct Solution (a) Recently, the research team of the Department of Wildlife Protection has confirmed the presence of snow leopard in Kishtwar High Altitude National Park through camera trap photographs. (Hence option a is correct) Camera traps had captured two snow leopards in Nant Nullah area. In Marwah and Dachhan areas, camera traps have captured images of three snow leopards. Important Points/Value Additions: Snow leopard listed as vulnerable on the International Union for Conservation of Nature red list in view of their depleting numbers. There are estimated to be fewer than 10,000 mature snow leopards across the globe. These animals usually live above the tree line on alpine meadows and in rocky regions at elevations of 2,700 metres to 6,000 metres during summer. In winter, they come down to elevations around 1,200 m to 2,000 m. Incorrect Solution (a) Recently, the research team of the Department of Wildlife Protection has confirmed the presence of snow leopard in Kishtwar High Altitude National Park through camera trap photographs. (Hence option a is correct) Camera traps had captured two snow leopards in Nant Nullah area. In Marwah and Dachhan areas, camera traps have captured images of three snow leopards. Important Points/Value Additions: Snow leopard listed as vulnerable on the International Union for Conservation of Nature red list in view of their depleting numbers. There are estimated to be fewer than 10,000 mature snow leopards across the globe. These animals usually live above the tree line on alpine meadows and in rocky regions at elevations of 2,700 metres to 6,000 metres during summer. In winter, they come down to elevations around 1,200 m to 2,000 m. Question 14 of 35 14. Question Consider following pairs: Protected Area                                     State Kaimur Wildlife Sanctury:             Bihar Yeslur Forest Range:                       Jharkhand Asola Bhatti Wildlife Sanctury:     Punjab How many of the above pairs are correct? a) Only one b) Only two c) All three d) None Correct Solution (a) The much-awaited second tiger reserve of Bihar is all set to come up either by the end of 2023 or early 2024, according to the officials of the Bihar Forest, Environment and Climate Change Department. The officials have been working towards obtaining the National Tiger Reserve Conservation Authority’s (NTCA) approval for declaring Kaimur Wildlife Sanctuary as the state’s second tiger reserve after the Valmiki Tiger Reserve (VTR). (Hence option 1 is correct) The state of Karnataka’s beloved elephant Arjuna was killed in a freak encounter with a wild tusker during an elephant capture operation in the Yeslur forest range in Hassan district. The operation was part of a mass radio collaring of elephants as per an order by the forest department. (Hence option 2 is incorrect) Restraining the Delhi government’s Forest department from holding a proposed event ‘Walk with Wildlife’ inside the city’s Asola Bhatti Wildlife Sanctuary, the Delhi High Court said that the department gave permission for the event in a mechanical manner. It added that the department did not analyse the threat perception as the event has trappings of a misadventure. The high court in its order also remarked that while wildlife sanctuaries are made for the preservation and protection of wildlife in their natural habitats, “lately, man has been encroaching on the habitat of wildlife”. (Hence option 3 is incorrect) Incorrect Solution (a) The much-awaited second tiger reserve of Bihar is all set to come up either by the end of 2023 or early 2024, according to the officials of the Bihar Forest, Environment and Climate Change Department. The officials have been working towards obtaining the National Tiger Reserve Conservation Authority’s (NTCA) approval for declaring Kaimur Wildlife Sanctuary as the state’s second tiger reserve after the Valmiki Tiger Reserve (VTR). (Hence option 1 is correct) The state of Karnataka’s beloved elephant Arjuna was killed in a freak encounter with a wild tusker during an elephant capture operation in the Yeslur forest range in Hassan district. The operation was part of a mass radio collaring of elephants as per an order by the forest department. (Hence option 2 is incorrect) Restraining the Delhi government’s Forest department from holding a proposed event ‘Walk with Wildlife’ inside the city’s Asola Bhatti Wildlife Sanctuary, the Delhi High Court said that the department gave permission for the event in a mechanical manner. It added that the department did not analyse the threat perception as the event has trappings of a misadventure. The high court in its order also remarked that while wildlife sanctuaries are made for the preservation and protection of wildlife in their natural habitats, “lately, man has been encroaching on the habitat of wildlife”. (Hence option 3 is incorrect) Question 15 of 35 15. Question Consider the following statements with respect to the recently released report titled ‘Elephant Corridors of India, 2023’: It mentioned an increase of 40 per cent of elephant corridors since 2010. Karnataka has the highest number of identified elephant corridors in India, accounting for over 17% of all the reported elephant corridors in the country. The Southern region harbors the largest elephant population in India. How many of the above statements are correct? a) Only one b) Only two c) All three d) None Correct Solution (b) A total of 150 elephant corridors were reported from 15 elephant range states across the four elephant-bearing regions of India. Elephant Task Force of the Government of India listed 88 corridors in to the Gajah report published during 2010. (Hence statement 1 is correct) West Bengal has the highest number of identified elephant corridors in India, accounting for over 17% of all the reported elephant corridors in the country. (Hence statement 2 is incorrect) Among the four elephant-bearing regions, nearly 35% of the elephant corridors were in the East-central region, followed by 32% in the North-east region. The Southern region, which harbors the largest elephant population in India accounts for 21% of the elephant corridors in India. (Hence statement 3 is correct) The Northern region that harbors the smallest of the four regional elephant populations, has the least number of elephant corridors, accounting for 12% of all the reported elephant corridors in the country. Incorrect Solution (b) A total of 150 elephant corridors were reported from 15 elephant range states across the four elephant-bearing regions of India. Elephant Task Force of the Government of India listed 88 corridors in to the Gajah report published during 2010. (Hence statement 1 is correct) West Bengal has the highest number of identified elephant corridors in India, accounting for over 17% of all the reported elephant corridors in the country. (Hence statement 2 is incorrect) Among the four elephant-bearing regions, nearly 35% of the elephant corridors were in the East-central region, followed by 32% in the North-east region. The Southern region, which harbors the largest elephant population in India accounts for 21% of the elephant corridors in India. (Hence statement 3 is correct) The Northern region that harbors the smallest of the four regional elephant populations, has the least number of elephant corridors, accounting for 12% of all the reported elephant corridors in the country. Question 16 of 35 16. Question Consider the following statements about POEM platform of ISRO: It will help perform in-orbit experiments using the final, and otherwise discarded stage of Geosynchronous Satellite Launch Vehicle (GSLV). It has a dedicated Navigation Guidance and Control (NGC) system for attitude stabilization. It allows space agencies and private companies to design, develop and validate experimental orbital payloads. How many of the above statements are correct? a) Only one b) Only two c) All three d) None Correct Solution (b) POEM (PSLV Orbital Experimental Module) is an experimental mission by ISRO which performs in-orbit scientific experiments during the fourth stage of the Polar Satellite Launch Vehicle (PSLV) launch vehicle as an orbital platform. (Hence statement 1 is incorrect) According to ISRO, POEM has a dedicated Navigation Guidance and Control (NGC) system for attitude stabilisation, which stands for controlling the orientation of any aerospace vehicle within permitted limits. The NGC will act as the platform’s brain to stabilize it with specified accuracy. (Hence statement 2 is correct) The space agency’s POEM platform has standard interfaces and packaging for power generation, telemetry, tele-command stabilisation, orbital-station keeping and orbital maneuvering. This allows space agencies and private companies to design, develop and validate experimental orbital payloads. (Hence statement 3 is correct) Incorrect Solution (b) POEM (PSLV Orbital Experimental Module) is an experimental mission by ISRO which performs in-orbit scientific experiments during the fourth stage of the Polar Satellite Launch Vehicle (PSLV) launch vehicle as an orbital platform. (Hence statement 1 is incorrect) According to ISRO, POEM has a dedicated Navigation Guidance and Control (NGC) system for attitude stabilisation, which stands for controlling the orientation of any aerospace vehicle within permitted limits. The NGC will act as the platform’s brain to stabilize it with specified accuracy. (Hence statement 2 is correct) The space agency’s POEM platform has standard interfaces and packaging for power generation, telemetry, tele-command stabilisation, orbital-station keeping and orbital maneuvering. This allows space agencies and private companies to design, develop and validate experimental orbital payloads. (Hence statement 3 is correct) Question 17 of 35 17. Question ‘Wetland City Accreditation (WCA)’ was established by which one of the following organizations? a) Wetlands International b) Ramsar Convention c) UN Environment Programme d) World Wildlife Fund Correct Solution (b) Wetlands International is a non-profit organization that works to restore and sustain wetlands and their resources for biodiversity and people. It is the only global organization dedicated to this purpose. (Option a is incorrect) The Wetland City Accreditation (WCA) is a voluntary program that recognizes cities that have made exceptional efforts to protect their urban wetlands. The Ramsar Convention established the WCA during the 2015 Conference of the Contracting Parties (COP) 12, and the accreditation is valid for six years. (Option b is correct) The United Nations Environment Programme (UNEP) is the United Nations’ leading environmental authority. It was established in 1972 by Maurice Strong and is responsible for coordinating responses to environmental issues within the United Nations system. (Option c is incorrect) Incorrect Solution (b) Wetlands International is a non-profit organization that works to restore and sustain wetlands and their resources for biodiversity and people. It is the only global organization dedicated to this purpose. (Option a is incorrect) The Wetland City Accreditation (WCA) is a voluntary program that recognizes cities that have made exceptional efforts to protect their urban wetlands. The Ramsar Convention established the WCA during the 2015 Conference of the Contracting Parties (COP) 12, and the accreditation is valid for six years. (Option b is correct) The United Nations Environment Programme (UNEP) is the United Nations’ leading environmental authority. It was established in 1972 by Maurice Strong and is responsible for coordinating responses to environmental issues within the United Nations system. (Option c is incorrect) Question 18 of 35 18. Question Consider the following points with reference to Namami Gange Programme:  Sewerage Treatment Infrastructure River-Front Development River-Surface Cleaning Ganga Gram How many of the points above are part of the main pillars of the Namami Gange Programme? a) Only one b) Only two c) Only three d) All four Correct Solution (d) ‘Namami Gange Programme’, is an Integrated Conservation Mission, approved as ‘Flagship Programme’ by the Union Government in June 2014 with budget outlay of Rs.20,000 Crore to accomplish the twin objectives of effective abatement of pollution, conservation and rejuvenation of National River Ganga. Sewerage Treatment Infrastructure is a core aspect of the Namami Gange Programme, aiming to enhance the capacity and efficiency of sewage treatment across the Ganga basin to reduce the direct discharge of untreated sewage into the river. (Statement 1 is correct) River-front development projects, including ghats and crematoria, are integral to the Namami Gange Programme. These projects aim to improve cleanliness, aesthetics, and environmental sustainability along the riverbanks. (Statement 2 is correct) River-Surface Cleaning involves removing floating waste from the surface of the Ganga and its tributaries. The initiative is crucial for improving water quality and maintaining the ecological health of the river. (Statement 3 is correct) The Ganga Gram initiative focuses on the holistic development of villages along the Ganges with improved sanitation, solid and liquid waste management, and water conservation practices. (Statement 4 is correct) Incorrect Solution (d) ‘Namami Gange Programme’, is an Integrated Conservation Mission, approved as ‘Flagship Programme’ by the Union Government in June 2014 with budget outlay of Rs.20,000 Crore to accomplish the twin objectives of effective abatement of pollution, conservation and rejuvenation of National River Ganga. Sewerage Treatment Infrastructure is a core aspect of the Namami Gange Programme, aiming to enhance the capacity and efficiency of sewage treatment across the Ganga basin to reduce the direct discharge of untreated sewage into the river. (Statement 1 is correct) River-front development projects, including ghats and crematoria, are integral to the Namami Gange Programme. These projects aim to improve cleanliness, aesthetics, and environmental sustainability along the riverbanks. (Statement 2 is correct) River-Surface Cleaning involves removing floating waste from the surface of the Ganga and its tributaries. The initiative is crucial for improving water quality and maintaining the ecological health of the river. (Statement 3 is correct) The Ganga Gram initiative focuses on the holistic development of villages along the Ganges with improved sanitation, solid and liquid waste management, and water conservation practices. (Statement 4 is correct) Question 19 of 35 19. Question Which one of the following best describes the process of “coal gasification”? a) A process that converts fossil-based carbonaceous materials into carbon monoxide, hydrogen, and carbon dioxide. b) The extraction of methane gas from coal seams without converting the coal into synthetic gas. c) A technique that prevents carbon dioxide from coal combustion from entering the atmosphere. d) The direct combustion of coal in power plants to generate electricity without any intermediate process. Correct Solution (a) Coal gasification is a process that involves the conversion of coal or any other carbonaceous material into synthetic gas (syngas), which primarily consists of carbon monoxide and hydrogen. The syngas produced can then be used for various applications, including electricity generation, production of chemical feedstocks, and as a fuel for heating. This process allows for more efficient use of coal and can reduce environmental impact compared to direct combustion. (Option a is correct) The statement describes the process of coal bed methane extraction, not coal gasification. Coal bed methane extraction involves the removal of methane gas directly from coal seams, which is distinct from the gasification process that produces synthetic gas from coal. (Option b is incorrect) This option refers to carbon capture and storage (CCS), a technology aimed at capturing carbon dioxide emissions from sources like power plants and storing it underground to prevent it from entering the atmosphere. While CCS can be associated with coal gasification, it is not a description of the gasification process itself. (Option c is incorrect) This option describes the traditional method of using coal for energy production through direct combustion. Coal gasification, however, involves a conversion process to produce syngas before any combustion for energy production takes place, making this process more versatile and potentially more environmentally friendly than direct combustion. (Option d is incorrect) Incorrect Solution (a) Coal gasification is a process that involves the conversion of coal or any other carbonaceous material into synthetic gas (syngas), which primarily consists of carbon monoxide and hydrogen. The syngas produced can then be used for various applications, including electricity generation, production of chemical feedstocks, and as a fuel for heating. This process allows for more efficient use of coal and can reduce environmental impact compared to direct combustion. (Option a is correct) The statement describes the process of coal bed methane extraction, not coal gasification. Coal bed methane extraction involves the removal of methane gas directly from coal seams, which is distinct from the gasification process that produces synthetic gas from coal. (Option b is incorrect) This option refers to carbon capture and storage (CCS), a technology aimed at capturing carbon dioxide emissions from sources like power plants and storing it underground to prevent it from entering the atmosphere. While CCS can be associated with coal gasification, it is not a description of the gasification process itself. (Option c is incorrect) This option describes the traditional method of using coal for energy production through direct combustion. Coal gasification, however, involves a conversion process to produce syngas before any combustion for energy production takes place, making this process more versatile and potentially more environmentally friendly than direct combustion. (Option d is incorrect) Question 20 of 35 20. Question Chemical Oxygen Demand (COD) is primarily used as a measure of –  a) The concentration of oxygen dissolved in freshwater bodies. b) The oxygen consumption rate of microorganisms in soil. c) The efficiency of oxygen usage in aerobic exercise. d) The total quantity of oxygen required to decompose organic and inorganic matter in water. Correct Solution (d) Chemical Oxygen Demand (COD) is an important environmental parameter used to assess the pollution level in water bodies. This option describes dissolved oxygen (DO), which measures the amount of oxygen freely available in water bodies, crucial for the survival of aquatic life. It does not relate to the COD, which focuses on the oxygen demand of water pollutants. (Option a is incorrect) Oxygen consumption rate of microorganisms in soil relates to soil respiration, a process different from COD. COD pertains specifically to water bodies and is unrelated to soil microbial activity. (Option b is incorrect) This option is related to the physiological measurement of oxygen efficiency during physical activities and is unrelated to COD, which is an environmental metric for water quality. (Option c is incorrect) Chemical Oxygen Demand (COD) measures the total quantity of oxygen required to chemically oxidize organic and inorganic pollutants in water. It is a critical parameter for assessing the pollution level of water, indicating the amount of oxygen needed to break down pollutants present. (Option d is correct) Incorrect Solution (d) Chemical Oxygen Demand (COD) is an important environmental parameter used to assess the pollution level in water bodies. This option describes dissolved oxygen (DO), which measures the amount of oxygen freely available in water bodies, crucial for the survival of aquatic life. It does not relate to the COD, which focuses on the oxygen demand of water pollutants. (Option a is incorrect) Oxygen consumption rate of microorganisms in soil relates to soil respiration, a process different from COD. COD pertains specifically to water bodies and is unrelated to soil microbial activity. (Option b is incorrect) This option is related to the physiological measurement of oxygen efficiency during physical activities and is unrelated to COD, which is an environmental metric for water quality. (Option c is incorrect) Chemical Oxygen Demand (COD) measures the total quantity of oxygen required to chemically oxidize organic and inorganic pollutants in water. It is a critical parameter for assessing the pollution level of water, indicating the amount of oxygen needed to break down pollutants present. (Option d is correct) Question 21 of 35 21. Question Consider the following statements about Pompeii It lies at the southeastern base of Mount Vesuvius. It is a preserved ancient Roman city located near modern-day Naples, Italy. It was built on a spur formed by a prehistoric lava flow to the north of the mouth of the Sarno River. It featured a complex municipal water system, an amphitheatre, and a gymnasium. How many of the above statements are correct? a) Only one b) Only two c) Only three d) All four Correct Solution (d) Pompeii lies at the southeastern base of Mount Vesuvius. Hence statement 1 is correct. It was conquered and absorbed into the Roman Empire in 80 BCE. At its height, the small city had a thriving economy based on trade and agriculture. The population of the town has been estimated at 10-12,000, with one-third being slaves. It is a preserved ancient Roman city located near modern-day Naples, Italy. Hence statement 2 is correct. The area was originally settled in the Bronze Age on an escarpment at the mouth of the Sarnus(modern Sarno) River. It was built on a spur formed by a prehistoric lava flow to the north of the mouth of the Sarno River. Hence statement 3 is correct. The city was buried in volcanic ash following the eruption of Mt. Vesuvius in 79 CE. Pompeii was lost and forgotten until it was rediscovered in 1748. Its excellent state of preservation gives an invaluable insight into Roman everyday life. It is a UNESCO World Heritage Site. It featured a complex municipal water system, an amphitheatre, and a gymnasium. Hence statement 4 is correct.   Incorrect Solution (d) Pompeii lies at the southeastern base of Mount Vesuvius. Hence statement 1 is correct. It was conquered and absorbed into the Roman Empire in 80 BCE. At its height, the small city had a thriving economy based on trade and agriculture. The population of the town has been estimated at 10-12,000, with one-third being slaves. It is a preserved ancient Roman city located near modern-day Naples, Italy. Hence statement 2 is correct. The area was originally settled in the Bronze Age on an escarpment at the mouth of the Sarnus(modern Sarno) River. It was built on a spur formed by a prehistoric lava flow to the north of the mouth of the Sarno River. Hence statement 3 is correct. The city was buried in volcanic ash following the eruption of Mt. Vesuvius in 79 CE. Pompeii was lost and forgotten until it was rediscovered in 1748. Its excellent state of preservation gives an invaluable insight into Roman everyday life. It is a UNESCO World Heritage Site. It featured a complex municipal water system, an amphitheatre, and a gymnasium. Hence statement 4 is correct.   Question 22 of 35 22. Question Consider the following statements about Bumblebees They are large, hairy, social bees belonging to the family Apidae. They occur throughout much of the world but are most common in tropical climates. They make honey like honeybees as they need to store food for winter. How many of the above statements are correct? a) Only one b) Only two c) All three d) None Correct Solution (a) Bumblebees are large, hairy, social bees belonging to the family Apidae. Hence statement 1 is correct. The family Apidae includes the well-known honey bees and bumble bees, as well as carpenter bees, cuckoo bees, digger bees, stingless bees, and orchid bees. They are large, yellow, and black flying insects with a distinct buzz. There is variation in coloration among bumblebees, and some species have bands of red, yellow, and black. They have stocky bodies that are covered with many hairs to which pollen adheres. They occur throughout much of the world but are most common in temperate climates. Hence statement 2 is incorrect. There are over 250 known species, existing primarily in the Northern Hemisphere. They are known for their ability to collect nectar from flowers and pollinate plants. Bumblebees have four wings; the two rear wings are small and usually attached to the fore wings by a row of hooks called hamuli. The wings move rapidly, at 130-240 beats per second. They are social insects that live in colonies. The queen bee, drones, and worker bees all have specific tasks to help support the colony. Unlike the honeybee, bumblebees do not make honey, as they do not need to store food for winter. Instead, the season’s new queens hibernate and emerge to find their nests in the spring. Hence statement 3 is incorrect. Incorrect Solution (a) Bumblebees are large, hairy, social bees belonging to the family Apidae. Hence statement 1 is correct. The family Apidae includes the well-known honey bees and bumble bees, as well as carpenter bees, cuckoo bees, digger bees, stingless bees, and orchid bees. They are large, yellow, and black flying insects with a distinct buzz. There is variation in coloration among bumblebees, and some species have bands of red, yellow, and black. They have stocky bodies that are covered with many hairs to which pollen adheres. They occur throughout much of the world but are most common in temperate climates. Hence statement 2 is incorrect. There are over 250 known species, existing primarily in the Northern Hemisphere. They are known for their ability to collect nectar from flowers and pollinate plants. Bumblebees have four wings; the two rear wings are small and usually attached to the fore wings by a row of hooks called hamuli. The wings move rapidly, at 130-240 beats per second. They are social insects that live in colonies. The queen bee, drones, and worker bees all have specific tasks to help support the colony. Unlike the honeybee, bumblebees do not make honey, as they do not need to store food for winter. Instead, the season’s new queens hibernate and emerge to find their nests in the spring. Hence statement 3 is incorrect. Question 23 of 35 23. Question Consider the following statements about Sudan It is mainly composed of vast plains and plateaus that are drained by the Nile River and its tributaries. It shares a border only with its five neighbours South Sudan, Egypt, Libya, Chad, and the Central African Republic. Khartoum is its capital and its official languages are Arabic and English. How many of the above statements are correct? a) Only one b) Only two c) All three d) None Correct Solution (b) Sudan is mainly composed of vast plains and plateaus that are drained by the Nile River and its tributaries. Hence statement 1 is correct. Much of Sudan consists of deserts and arid grasslands with little in the way of vegetation. The northern part of this plain is a primarily rocky desert area called the Nubian Desert. It is located in northeastern Africa. It is the third largest nation in all of Africa occupying an area of 1,886,068 sq. km. It shares a border only with its seven neighbours South Sudan, Ethiopia, Eritrea, Egypt, Libya, Chad, and the Central African Republic. Hence statement 2 is incorrect. Sudan also has a significant coastline along the Red Sea. This narrow strip of water gives vital access to the Indian Ocean as well as the Mediterranean Sea through the Suez Canal. Before the secession of the south in 2011, Sudan was the largest African country, with an area that represented more than 8 percent of the African continent It is dominated by Muslims who identify themselves as Arabs. Khartoum is its capital and its official languages are Arabic and English. Hence statement 3 is correct. Incorrect Solution (b) Sudan is mainly composed of vast plains and plateaus that are drained by the Nile River and its tributaries. Hence statement 1 is correct. Much of Sudan consists of deserts and arid grasslands with little in the way of vegetation. The northern part of this plain is a primarily rocky desert area called the Nubian Desert. It is located in northeastern Africa. It is the third largest nation in all of Africa occupying an area of 1,886,068 sq. km. It shares a border only with its seven neighbours South Sudan, Ethiopia, Eritrea, Egypt, Libya, Chad, and the Central African Republic. Hence statement 2 is incorrect. Sudan also has a significant coastline along the Red Sea. This narrow strip of water gives vital access to the Indian Ocean as well as the Mediterranean Sea through the Suez Canal. Before the secession of the south in 2011, Sudan was the largest African country, with an area that represented more than 8 percent of the African continent It is dominated by Muslims who identify themselves as Arabs. Khartoum is its capital and its official languages are Arabic and English. Hence statement 3 is correct. Question 24 of 35 24. Question Consider the following statements about the Laccadive Sea It is a body of water bordering India, the Maldives, and Sri Lanka. It is part of the Indian Ocean, the world’s third-largest body of water. It touches the Nine Degree Channel to its north. How many of the above statements are correct? a) Only one b) Only two c) All three d) None Correct Solution (b) The Laccadive Sea is a body of water bordering India, the Maldives, and Sri Lanka. Hence statement 1 is correct. It is 303,476 square miles in area. It is part of the Indian Ocean, the world’s third-largest body of water. Hence statement 2 is correct. Lakshadweep Sea meets the Arabian Sea in the north, while to the northeast, it borders the Kerala state of India. It touches the Nine Degree Channel to its south. Hence statement 3 is incorrect. With an average depth of 6,329 feet, the Lakshadweep Sea is only half as deep as the average depth of the entire Indian Ocean. This warm sea has a stable water temperature throughout the year and is rich in marine life; the Gulf of Mannar alone hosts about 3,600 species. Considering the coral reefs of the Lakshadweep Sea alone, there are nearly 120 observed living species. Major cities along its shores include Trivandrum, Kochi, Quilon, Colombo, and Male. Incorrect Solution (b) The Laccadive Sea is a body of water bordering India, the Maldives, and Sri Lanka. Hence statement 1 is correct. It is 303,476 square miles in area. It is part of the Indian Ocean, the world’s third-largest body of water. Hence statement 2 is correct. Lakshadweep Sea meets the Arabian Sea in the north, while to the northeast, it borders the Kerala state of India. It touches the Nine Degree Channel to its south. Hence statement 3 is incorrect. With an average depth of 6,329 feet, the Lakshadweep Sea is only half as deep as the average depth of the entire Indian Ocean. This warm sea has a stable water temperature throughout the year and is rich in marine life; the Gulf of Mannar alone hosts about 3,600 species. Considering the coral reefs of the Lakshadweep Sea alone, there are nearly 120 observed living species. Major cities along its shores include Trivandrum, Kochi, Quilon, Colombo, and Male. Question 25 of 35 25. Question Consider the following statements about the Athletics Federation of India (AFI) It is a statutory top governing body responsible for overseeing athletics in India. It organises the National Championships and selects the Indian Athletics Teams for various international competitions. It is affiliated with the World Athletics, the Asian Athletics Association (AAA), and the Indian Olympic Association. How many of the above statements are correct? a) Only one b) Only two c) All three d) None Correct Solution (b) The Athletics Federation of India (AFI) is a non-governmental, non-profitable autonomous top governing body responsible for overseeing athletics in India. Hence statement 1 is incorrect. It was formerly called the Amateur Athletic Federation of India (AAFI). The AFI has as many as 32 affiliated state units and institutional units. It organises the National Championships and selects the Indian Athletics Teams for various international competitions. Hence statement 2 is correct. It trains the Indian Athletics National Campers, selects the Indian Athletics Teams for various international competitions, including the Olympics, Asian Games, CWG, World Championships, Asian Championships, and other international meets, and conducts the National Championships for various age categories. The AFI conducts international and national championships and various meets to promote the sport, popularise it amongst the masses, and make athletics commercially attractive for the further growth of the athlete and the sport. It supervises and assists its state units in their activities, plans and sets up special coaching camps, coaches training, and takes initiatives for development programme and grassroots promotion of athletics in India. It is affiliated with the World Athletics, the Asian Athletics Association (AAA), and the Indian Olympic Association. Hence statement 3 is correct.   Incorrect Solution (b) The Athletics Federation of India (AFI) is a non-governmental, non-profitable autonomous top governing body responsible for overseeing athletics in India. Hence statement 1 is incorrect. It was formerly called the Amateur Athletic Federation of India (AAFI). The AFI has as many as 32 affiliated state units and institutional units. It organises the National Championships and selects the Indian Athletics Teams for various international competitions. Hence statement 2 is correct. It trains the Indian Athletics National Campers, selects the Indian Athletics Teams for various international competitions, including the Olympics, Asian Games, CWG, World Championships, Asian Championships, and other international meets, and conducts the National Championships for various age categories. The AFI conducts international and national championships and various meets to promote the sport, popularise it amongst the masses, and make athletics commercially attractive for the further growth of the athlete and the sport. It supervises and assists its state units in their activities, plans and sets up special coaching camps, coaches training, and takes initiatives for development programme and grassroots promotion of athletics in India. It is affiliated with the World Athletics, the Asian Athletics Association (AAA), and the Indian Olympic Association. Hence statement 3 is correct.   Question 26 of 35 26. Question Which of the following factors causes Imported Inflation? Exchange rates Commodity prices Trade policies Transportation costs Choose the correct code: a) Only one b) Only two c) Only three d) All four Correct Solution (d) Imported inflation is a general and sustainable price increase due to an increase in the costs of imported products. Factors that cause Imported Inflation: The most significant driver of imported inflation is fluctuations in exchange rates. The more the currency depreciates on the foreign exchange market, the higher the price of imports. Effectively, more money is needed to buy goods and services outside the country. Hence statement 1 is correct. Many countries, particularly smaller countries, are highly dependent on imported commodities like oil, metals, and agricultural products. When commodity prices rise globally, it directly impacts the cost of imports and can lead to higher inflation in the importing country. Hence statement 2 is correct. Changes in trade policies, such as tariffs and quotas, can influence the cost of imported goods. Hence statement 3 is correct. Fluctuations in transportation costs, influenced by factors like fuel prices and logistical challenges, can affect the final cost of imported goods. Hence statement 4 is correct.   Incorrect Solution (d) Imported inflation is a general and sustainable price increase due to an increase in the costs of imported products. Factors that cause Imported Inflation: The most significant driver of imported inflation is fluctuations in exchange rates. The more the currency depreciates on the foreign exchange market, the higher the price of imports. Effectively, more money is needed to buy goods and services outside the country. Hence statement 1 is correct. Many countries, particularly smaller countries, are highly dependent on imported commodities like oil, metals, and agricultural products. When commodity prices rise globally, it directly impacts the cost of imports and can lead to higher inflation in the importing country. Hence statement 2 is correct. Changes in trade policies, such as tariffs and quotas, can influence the cost of imported goods. Hence statement 3 is correct. Fluctuations in transportation costs, influenced by factors like fuel prices and logistical challenges, can affect the final cost of imported goods. Hence statement 4 is correct.   Question 27 of 35 27. Question Consider the following statements about Ashwagandha It is an evergreen shrub native to India, Northern Africa, and the Middle East. It belongs to the Solanaceae family, the same family as tomatoes and potatoes. It is called an adaptogen as it helps the body adapt to stressors and restore balance. How many of the above statements are correct? a) Only one b) Only two c) All three d) None Correct Solution (c) Ashwagandha is an evergreen shrub native to India, Northern Africa, and the Middle East. Hence statement 1 is correct. It is scientifically known as Withania somnifera, the shrub got its name Ashwagandha because its roots are said to smell like a wet horse (‘ashwa’ for horse and ‘gandha’ for smell). It belongs to the Solanaceae family, the same family as tomatoes and potatoes. Hence statement 2 is correct. It has been in use as a medicinal plant for thousands of years, especially in traditional Ayurvedic medicine. It helps reduce inflammation, increase energy, alleviate anxiety, ease pain, and improve sleep. It is called an adaptogen as it helps the body adapt to stressors and restore balance. Hence statement 3 is correct. Incorrect Solution (c) Ashwagandha is an evergreen shrub native to India, Northern Africa, and the Middle East. Hence statement 1 is correct. It is scientifically known as Withania somnifera, the shrub got its name Ashwagandha because its roots are said to smell like a wet horse (‘ashwa’ for horse and ‘gandha’ for smell). It belongs to the Solanaceae family, the same family as tomatoes and potatoes. Hence statement 2 is correct. It has been in use as a medicinal plant for thousands of years, especially in traditional Ayurvedic medicine. It helps reduce inflammation, increase energy, alleviate anxiety, ease pain, and improve sleep. It is called an adaptogen as it helps the body adapt to stressors and restore balance. Hence statement 3 is correct. Question 28 of 35 28. Question Consider the following statements about Lake Kariba It is the world’s largest man-made lake and reservoir by volume. It lies upstream from the Indian Ocean, along the border between Zambia and Zimbabwe. Choose the correct code: a) 1 only b) 2 only c) Both 1 and 2 d) Neither 1 nor 2 Correct Solution (c) Lake Kariba is the world’s largest man-made lake and reservoir by volume. Hence statement 1 is correct. It is 200 kilometers downstream of Victoria Falls. The lake was filled following the completion of the Kariba Dam wall at its northeastern end, flooding the Kariba Gorge on the Zambezi River. The Kariba Dam consists of a double-arch wall. It is 128 meters in height, 617 meters in length, 13 meters wide at its top, and 24 meters wide at the base. The wall extends across the Kariba Gorge, creating a border crossing between Zambia and Zimbabwe. It took up to three years for the dam wall to be completed. Construction began on the 6th of November, 1956, and was finished in 1959. It provides considerable electric power to both Zambia and Zimbabwe and supports a thriving commercial fishing industry. It is roughly estimated to be about 280 km long and 40 km at its widest. It covers an area of nearly 6,000 square kilometres. It lies upstream from the Indian Ocean, along the border between Zambia and Zimbabwe. Hence statement 2 is correct. Incorrect Solution (c) Lake Kariba is the world’s largest man-made lake and reservoir by volume. Hence statement 1 is correct. It is 200 kilometers downstream of Victoria Falls. The lake was filled following the completion of the Kariba Dam wall at its northeastern end, flooding the Kariba Gorge on the Zambezi River. The Kariba Dam consists of a double-arch wall. It is 128 meters in height, 617 meters in length, 13 meters wide at its top, and 24 meters wide at the base. The wall extends across the Kariba Gorge, creating a border crossing between Zambia and Zimbabwe. It took up to three years for the dam wall to be completed. Construction began on the 6th of November, 1956, and was finished in 1959. It provides considerable electric power to both Zambia and Zimbabwe and supports a thriving commercial fishing industry. It is roughly estimated to be about 280 km long and 40 km at its widest. It covers an area of nearly 6,000 square kilometres. It lies upstream from the Indian Ocean, along the border between Zambia and Zimbabwe. Hence statement 2 is correct. Question 29 of 35 29. Question Consider the following statements about Tachyons They are hypothetical subatomic particles that move faster than the speed of light. They are familiar and include protons, electrons, and neutrons. Choose the correct code: a) 1 only b) 2 only c) Both 1 and 2 d) Neither 1 nor 2 Correct Solution (a) Tachyons are hypothetical subatomic particles that move faster than the speed of light. Hence statement 1 is correct. The term “tachyon” was coined by physicist Gerald Feinberg in 1967. They are distinguished from “bradyons,” particles that travel at less than the speed of light. While bradyons are familiar and include protons, electrons, and neutrons, tachyons have never been observed. Hence statement 2 is incorrect. Tachyons are thought to have imaginary mass, meaning their mass squared is a negative value. This implies that they could potentially travel faster than light without violating the laws of physics as we currently understand them. Tachyons would slow down if they gained energy, and accelerate if they lost energy. There have been a few experiments to find tachyons using a detector called a Cerenkov detector. This detector can measure the speed of a particle traveling through a medium. Incorrect Solution (a) Tachyons are hypothetical subatomic particles that move faster than the speed of light. Hence statement 1 is correct. The term “tachyon” was coined by physicist Gerald Feinberg in 1967. They are distinguished from “bradyons,” particles that travel at less than the speed of light. While bradyons are familiar and include protons, electrons, and neutrons, tachyons have never been observed. Hence statement 2 is incorrect. Tachyons are thought to have imaginary mass, meaning their mass squared is a negative value. This implies that they could potentially travel faster than light without violating the laws of physics as we currently understand them. Tachyons would slow down if they gained energy, and accelerate if they lost energy. There have been a few experiments to find tachyons using a detector called a Cerenkov detector. This detector can measure the speed of a particle traveling through a medium. Question 30 of 35 30. Question Consider the following statements regarding the State Emblem of India It was carved out of a single block of polished sandstone. The words Satyameva Jayate inscribed on the national emblem are taken from Mundaka Upanishad. Choose the correct code: a) 1 only b) 2 only c) Both 1 and 2 d) Neither 1 nor 2 Correct Solution (c) The State Emblem of India was carved out of a single block of polished sandstone. Hence statement 1 is correct. The state emblem of Indiais an adaptation of the Sarnath Lion Capital of Ashoka. In the original, there are four lions, standing back to back, mounted on an abacus with a frieze carrying sculptures in high relief of an elephant, a galloping horse, a bull, and a lion separated by intervening wheels over a bell-shaped lotus. The words Satyameva Jayate inscribed on the national emblem are taken from Mundaka Upanishad. Hence statement 2 is correct. Incorrect Solution (c) The State Emblem of India was carved out of a single block of polished sandstone. Hence statement 1 is correct. The state emblem of Indiais an adaptation of the Sarnath Lion Capital of Ashoka. In the original, there are four lions, standing back to back, mounted on an abacus with a frieze carrying sculptures in high relief of an elephant, a galloping horse, a bull, and a lion separated by intervening wheels over a bell-shaped lotus. The words Satyameva Jayate inscribed on the national emblem are taken from Mundaka Upanishad. Hence statement 2 is correct. Question 31 of 35 31. Question Segmenting the labour market and creating a separate labour market for migrant workers — who are easy to exploit — has been a common strategy of employers across India. The pathetic conditions migrant workers face have been widely documented. They earn low wages, work very long hours without any overtime benefits, and are almost without any leave or social protection. Lakhs of unskilled and migrant workers live on worksites in makeshift huts (usually made of tin sheets) or on roads, slums and in illegal settlements not served by municipalities. They are neither able to save much to improve their conditions back in their home States nor save enough to live comfortably in a migrating state. They go back home only once or twice to celebrate festivals. Semi-skilled workers with some education and skills (such as those in diamond cutting and polishing units, power looms and factories) get slightly higher wages and earn some leave. However, these workers are also exploited in multiple ways and are mostly unprotected. Factory owners, employers and traders are only too happy with such a situation as they earn huge profits from wage labour exploitation.   On the basis of the above passage, the following inferences have been made: More than 50% of the migrant workers in India are engaged in either unskilled or semi-skilled jobs. Semi-skilled workers with some education and skills, are not exploited as they earn huge profits for their employers. Which of the above statements is/are valid? a) 1 only b) 2 only c) Both 1 and 2 d) Neither 1 nor 2 Correct Solution (d) The passage mentions the poor conditions of unskilled and semi-skilled migrant workers. But there is no clarity in the passage about what proportion of migrant workers are unskilled and semi-skilled. Hence statement 1 is incorrect. The passage clearly mentions that though the wages of semi-skilled workers are slightly higher, they are also exploited by their employers. Therefore, it would be incorrect to infer that they are not exploited by their workers. Hence statement 2 is also INCORRECT. Hence the CORRECT answer is option ‘D’. Incorrect Solution (d) The passage mentions the poor conditions of unskilled and semi-skilled migrant workers. But there is no clarity in the passage about what proportion of migrant workers are unskilled and semi-skilled. Hence statement 1 is incorrect. The passage clearly mentions that though the wages of semi-skilled workers are slightly higher, they are also exploited by their employers. Therefore, it would be incorrect to infer that they are not exploited by their workers. Hence statement 2 is also INCORRECT. Hence the CORRECT answer is option ‘D’. Question 32 of 35 32. Question Segmenting the labour market and creating a separate labour market for migrant workers — who are easy to exploit — has been a common strategy of employers across India. The pathetic conditions migrant workers face have been widely documented. They earn low wages, work very long hours without any overtime benefits, and are almost without any leave or social protection. Lakhs of unskilled and migrant workers live on worksites in makeshift huts (usually made of tin sheets) or on roads, slums and in illegal settlements not served by municipalities. They are neither able to save much to improve their conditions back in their home States nor save enough to live comfortably in a migrating state. They go back home only once or twice to celebrate festivals. Semi-skilled workers with some education and skills (such as those in diamond cutting and polishing units, power looms and factories) get slightly higher wages and earn some leave. However, these workers are also exploited in multiple ways and are mostly unprotected. Factory owners, employers and traders are only too happy with such a situation as they earn huge profits from wage labour exploitation. What is the crucial message conveyed by the above passage? a) Labour laws in India are not being implemented properly. b) The migrant workers are being exploited by the employers across India. c) In the long term migrant workers are able to improve their life back home. d) Migrant labourers constitute majority of the workforce in India. Correct Solution (b) The passage does not delve into the details of labour laws. We do not know whether labour laws in India are being implemented properly or not. Hence option A is INCORRECT. The passage extensively talks about the exploitation of migrant workers by the employers across India. Hence, option B is correct. The passage only talks about the poor conditions of migrant workers. There is no mention about its improvement in the long term. Hence, statement C is INCORRECT. The passage only mentions that there is a separate labour market for migrant workers. There is no clear mention about the proportion of migrant workers in India’s total workforce. Hence, statement D is INCORRECT. Incorrect Solution (b) The passage does not delve into the details of labour laws. We do not know whether labour laws in India are being implemented properly or not. Hence option A is INCORRECT. The passage extensively talks about the exploitation of migrant workers by the employers across India. Hence, option B is correct. The passage only talks about the poor conditions of migrant workers. There is no mention about its improvement in the long term. Hence, statement C is INCORRECT. The passage only mentions that there is a separate labour market for migrant workers. There is no clear mention about the proportion of migrant workers in India’s total workforce. Hence, statement D is INCORRECT. Question 33 of 35 33. Question Families around the world look, feel, and live differently today. Families can be “make or break” for women and girls when it comes to achieving their rights. They can be places of love, care, and fulfilment but, too often, they are also spaces where women‘s and girls’ rights are violated, their voices are stifled, and where gender inequality prevails. In today‘s changing world, laws and policies need to be based on the reality of how families live. With reference to the above passage, the following assumptions have been made: Families can thrive in happiness and security at the expense of its women‘s rights. Families are universally patriarchal. Laws and regulations regarding family do not adequately address the concerns of women. Freedom of expression within family and family fulfilment are mutually exclusive goals for a woman. Which of the above assumptions is/are valid? a) 1 and 4 b) 2 and 4 c) 1 and 3 d) 1, 2 and 3 Correct Solution (c) Statement 1: Family is depicted as the place for love, care and fulfilment and also the place where women’s rights are subjugated. Hence this assumption is valid. Statement 2: The first line mentions that families around the world look and feel different today and that families can make or break a woman with respect to achieving rights. There is thus a possibility that families are not entirely patriarchal. Statement 3: The final sentence of the passage mentions that laws and policies need to be based on the reality of how families live. This implies that current laws are inadequate in dealing with issues of women’s rights. Statement 4: According to the passage, gender inequality exists within many families and suppression of Women’s expression is one of the manifestations. However, it does not imply that securing women’s rights to express her opinion would run contrary to the goal of family fulfilment. Both should be possible and that is what laws and policies should aim for. Incorrect Solution (c) Statement 1: Family is depicted as the place for love, care and fulfilment and also the place where women’s rights are subjugated. Hence this assumption is valid. Statement 2: The first line mentions that families around the world look and feel different today and that families can make or break a woman with respect to achieving rights. There is thus a possibility that families are not entirely patriarchal. Statement 3: The final sentence of the passage mentions that laws and policies need to be based on the reality of how families live. This implies that current laws are inadequate in dealing with issues of women’s rights. Statement 4: According to the passage, gender inequality exists within many families and suppression of Women’s expression is one of the manifestations. However, it does not imply that securing women’s rights to express her opinion would run contrary to the goal of family fulfilment. Both should be possible and that is what laws and policies should aim for. Question 34 of 35 34. Question Statements followed by Conclusions are given below. You have to take the statements to be true even if they seem to be at variance from the commonly known facts. Read all the conclusions and then decide which of the given conclusions logically follows/follow from the statements, disregarding the commonly known facts: Statements: Some ants are parrots. All the parrots are apples.   Conclusions: All the apples are parrots. Some ants are apples. Choose the correct code: a) Only (1) conclusion follows b) Only (2) conclusion follows c) Either (1) or (2) follows d) Neither (1) nor (2) follows Correct Solution (b) Incorrect Solution (b) Question 35 of 35 35. Question Statements followed by Conclusions are given below. You have to take the statements to be true even if they seem to be at variance from the commonly known facts. Read all the conclusions and then decide which of the given conclusions logically follows/follow from the statements, disregarding the commonly known facts: Statements: All Trucks are Cars All Vans are Cars No Bike is Van. Conclusions: Some Bike is Car All Truck are Bikes Some Cars are not Bikes Choose the correct code: a) Only II and III b) Only III c) None of the conclusions follow d) I, II and III Correct Solution (b) Here, as shown in the first figure, the set of Trucks and Bikes can be the same only in one case but not always. In this case, conclusion II definitely does not follow. In the second figure, no car is a bike. Hence, conclusion I does not follow. Also, some Cars are not Bikes implies at least one car is not a bike. Since no bike is a van and all vans are cars, any car that is a van cannot be a bike. Hence, some cars are definitely not bikes. Hence, conclusion III follows. Hence, only conclusion III follows. Hence, option b. Incorrect Solution (b) Here, as shown in the first figure, the set of Trucks and Bikes can be the same only in one case but not always. In this case, conclusion II definitely does not follow. In the second figure, no car is a bike. Hence, conclusion I does not follow. Also, some Cars are not Bikes implies at least one car is not a bike. Since no bike is a van and all vans are cars, any car that is a van cannot be a bike. Hence, some cars are definitely not bikes. Hence, conclusion III follows. Hence, only conclusion III follows. Hence, option b. window.wpProQuizInitList = window.wpProQuizInitList || []; window.wpProQuizInitList.push({ id: '#wpProQuiz_3731', init: { quizId: 3731, mode: 1, globalPoints: 70, timelimit: 1800, resultsGrade: [0], bo: 704, qpp: 0, catPoints: [70], formPos: 0, lbn: "Test-summary", json: {"34067":{"type":"single","id":34067,"catId":0,"points":2,"correct":[0,0,0,1]},"34068":{"type":"single","id":34068,"catId":0,"points":2,"correct":[0,1,0,0]},"34071":{"type":"single","id":34071,"catId":0,"points":2,"correct":[1,0,0,0]},"34074":{"type":"single","id":34074,"catId":0,"points":2,"correct":[0,0,0,1]},"34078":{"type":"single","id":34078,"catId":0,"points":2,"correct":[0,1,0,0]},"34081":{"type":"single","id":34081,"catId":0,"points":2,"correct":[0,1,0,0]},"34095":{"type":"single","id":34095,"catId":0,"points":2,"correct":[0,1,0,0]},"34097":{"type":"single","id":34097,"catId":0,"points":2,"correct":[0,1,0,0]},"34100":{"type":"single","id":34100,"catId":0,"points":2,"correct":[0,0,1,0]},"34102":{"type":"single","id":34102,"catId":0,"points":2,"correct":[0,1,0,0]},"34104":{"type":"single","id":34104,"catId":0,"points":2,"correct":[0,0,1,0]},"34107":{"type":"single","id":34107,"catId":0,"points":2,"correct":[1,0,0,0]},"34110":{"type":"single","id":34110,"catId":0,"points":2,"correct":[1,0,0,0]},"34112":{"type":"single","id":34112,"catId":0,"points":2,"correct":[1,0,0,0]},"34114":{"type":"single","id":34114,"catId":0,"points":2,"correct":[0,1,0,0]},"34117":{"type":"single","id":34117,"catId":0,"points":2,"correct":[0,1,0,0]},"34120":{"type":"single","id":34120,"catId":0,"points":2,"correct":[0,1,0,0]},"34121":{"type":"single","id":34121,"catId":0,"points":2,"correct":[0,0,0,1]},"34124":{"type":"single","id":34124,"catId":0,"points":2,"correct":[1,0,0,0]},"34127":{"type":"single","id":34127,"catId":0,"points":2,"correct":[0,0,0,1]},"34130":{"type":"single","id":34130,"catId":0,"points":2,"correct":[0,0,0,1]},"34133":{"type":"single","id":34133,"catId":0,"points":2,"correct":[1,0,0,0]},"34134":{"type":"single","id":34134,"catId":0,"points":2,"correct":[0,1,0,0]},"34136":{"type":"single","id":34136,"catId":0,"points":2,"correct":[0,1,0,0]},"34139":{"type":"single","id":34139,"catId":0,"points":2,"correct":[0,1,0,0]},"34142":{"type":"single","id":34142,"catId":0,"points":2,"correct":[0,0,0,1]},"34144":{"type":"single","id":34144,"catId":0,"points":2,"correct":[0,0,1,0]},"34145":{"type":"single","id":34145,"catId":0,"points":2,"correct":[0,0,1,0]},"34148":{"type":"single","id":34148,"catId":0,"points":2,"correct":[1,0,0,0]},"34150":{"type":"single","id":34150,"catId":0,"points":2,"correct":[0,0,1,0]},"34152":{"type":"single","id":34152,"catId":0,"points":2,"correct":[0,0,0,1]},"34154":{"type":"single","id":34154,"catId":0,"points":2,"correct":[0,1,0,0]},"34155":{"type":"single","id":34155,"catId":0,"points":2,"correct":[0,0,1,0]},"34157":{"type":"single","id":34157,"catId":0,"points":2,"correct":[0,1,0,0]},"34159":{"type":"single","id":34159,"catId":0,"points":2,"correct":[0,1,0,0]}} } }); All the Best IASbaba

DAILY CURRENT AFFAIRS IAS | UPSC Prelims and Mains Exam –11th May 2024

Archives (PRELIMS & MAINS Focus)   Export-import in agri sector Syllabus Mains – GS 2 & GS 3 Context: India’s agricultural exports fell 8.2% in the fiscal year ended March 31, 2024 on the back of shipment curbs on a host of commodities, from cereals and sugar to onions. Background:- The value of farm exports totaled $48.82 billion in 2023-24, down from the record $53.15 billion of 2022-23 and $50.24 billion for the previous fiscal, according to Department of Commerce data. Key takeaways Exports declined during the initial years of the Narendra Modi government (from $43.25 billion in 2013-14 to $35.60 billion in 2019-20), while accompanied by an increase in imports (from $15.53 billion to $21.86 billion). That was largely because of a crash in global agri-commodity prices. Low international prices reduced the cost competitiveness of the country’s exports, while also making it more vulnerable to imports. But the global price recovery following the Covid-19 pandemic and Russia’s invasion of Ukraine resulted in India’s farm exports as well as imports zooming to all-time-highs in 2022-23, before dropping in the fiscal just ended. Drivers of exports: The fall in exports to have been led primarily by sugar and non-basmati rice.The government hasn’t allowed any sugar to go out of the country during the current production year from October 2023. Concerns over domestic availability and food inflation have similarly triggered a ban on exports of all white non-basmati rice since July 2023. Currently, only parboiled grain shipments are being permitted within the non-basmati segment, while also attracting a 20% duty. Two other items that have borne the brunt of export restrictions — again triggered by domestic shortages and rising prices — are wheat and onion. Drivers of imports: There is 7.9% dip in overall agri imports during 2023-24 due to a single commodity: edible oils. Lower global prices, in turn, brought down the vegetable oil import bill to below $15 billion during last fiscal. But even as the foreign exchange outflow on account of cooking oil has reduced, imports of pulses almost doubled to $3.75 billion in 2023-24, the highest since the $3.90 billion and $4.24 billion levels of 2015-16 and 2016-17 respectively. Policy takeaways Farmers and agri-traders, like all businessmen, want policy stability and predictability. When governments resort to banning/restricting agri export they usually privilege the interests of consumers over producers. These actions hurt more when taken overnight, like with wheat exports. Building export markets takes time and effort. A more predictable and rules-based policy — say, introducing temporary tariffs instead of outright bans or quantitative restrictions — is what many economists would recommend. The same goes for imports. The Modi government has done away with import duties on most pulses — arhar (pigeon pea), urad (black gram), masoor (red lentils), yellow/white peas and, earlier this month, chana (chickpea) — and kept it at 5.5% for crude palm, soyabean and sunflower oil. The above zero/low tariffs are at variance with the government’s own objective to promote crop diversification — weaning away farmers from rice, wheat and sugarcane to growing pulses and oilseeds, which are less water-guzzling and also significantly imported. Source:Indian Express Air Pollution Syllabus Prelims & Mains – Environment Context: Environmental, climate change, and air pollution issues have featured in the 2024 Lok Sabha manifestos of most top political parties. But is it among the top priorities or guarantees for parties or candidates? This brings us to another question: Will we ever witness actual improvement in air quality without it becoming a people’s movement or a political issue? Background: Despite mention in manifestos, pollution hasn’t become a campaign issue. It is a reflection of low traction at grassroots level. Key Takeaways According to a 2019 study, the yearly deaths attributable to air pollution translate to an economic loss of Rs 2.7 lakh crore, that is, around 1.36 per cent of the country’s GDP. Another recent survey has revealed that the Indian GDP would have been 4.5 per cent higher if air pollution had grown 50 per cent slower each year. What needs to be done to tackle air pollution? Any effective step to tackle pollution will come only when the issue becomes a mass movement. What it needs is public awareness in its true sense.It is the job of academicians, scientific communities, experts, scholars, and bureaucrats alongside local governmental bodies to make common Indians realise that clean air is also a fundamental right like clean water, health, food, shelter, etc. In addition to core research and scientific activities, a significant portion of the fund should be allocated for on-ground activities aimed at pollution reduction. Activities to tackle pollution need a federal structure, where policies and strategies need to be decentralised and diffused into micro environments through district and local bodies. Every ward under the municipalities or municipal corporations and every village under the blocks should be thoroughly scrutinised by the respective local bodies to find out the pollution source in the vicinity as well as the scope for air quality improvement. This information should then be disseminated to the people who are residents of the area. There should be specific plans to identify open areas favourable for the ventilation of air — and hence the pollutants — open water bodies, green cover for every ward in a city, and all of them should immediately be marked as green zones and restored. These measures will ensure that air pollution, as an issue, directly connects to every single individual in these micro environments. Regular outreach or public awareness programmes should be conducted at the municipality or block level, and facilitated by local experts, academicians, and teachers. These initiatives must aim to disseminate knowledge about environmental pollution and provide guidance on both actions to take and actions to avoid. Source: Indian Express NON-MARKET ECONOMY (NME) Syllabus Prelims – Economy Context: Vietnam has been actively advocating for a shift from its current “non-market economy” status to a “market economy” designation in the United States. Background: For over two decades, Vietnam has remained on Washington’s list of non-market economies. By achieving market economy status, Vietnam hopes to avoid high import taxes and enhance its trade prospects. However, this shift involves complex negotiations and considerations, including opposition from certain US industries. Key takeaways: A non-market economy refers to a country where market forces do not play a dominant role in determining prices, wages, and resource allocation. In contrast to market economies, where supply and demand largely dictate economic decisions. Non-market economies exhibit distinct characteristics. Characteristics of Non-Market Economies Currency Convertibility: Non-market economies may have restrictions on currency convertibility. Their currencies might not be freely traded on international markets. Wage Determination: Wages in non-market economies are often set through centralized mechanisms (such as government or industry agreements) rather than free bargaining between labour and management. Ownership of Means of Production: The state often owns or significantly controls major industries and resources. Resource Allocation and Price Decisions: The government plays a central role in allocating resources and making decisions related to prices and output. Human Rights Considerations: Some assessments also take into account human rights practices within a country when determining its economic status. Implications of Non-Market Economy Status: Countries classified as non-market economies face specific challenges in international trade: Anti-Dumping Measures: Non-market economies are subject to anti-dumping duties imposed by other countries. Anti-dumping measures aim to prevent unfair trade practices, such as selling goods below their production cost to gain an advantage in foreign markets. Trade Disputes: Disputes often arise between non-market economies and their trading partners regarding fair pricing, subsidies, and market access. Trade Relations: Non-market economy status affects a country’s trade relations, investment climate, and access to global markets. Source: Indian Express SHINKU LA TUNNEL Syllabus Prelims – Geography Context: The Border Roads Organisation will construct the world’s highest tunnel at Shinku La Pass at 16,580 feet to connect Himachal to Zanskar valley in Ladakh. Background: The developments come at a time when the military standoff between India and China in eastern Ladakh has entered its fifth year, with no indication of a resolution to the outstanding problems along the contested Line of Actual Control (LAC). About SHINKU LA TUNNEL : The Shinku La tunnel is a significant infrastructure project aimed at enhancing connectivity in the border areas of Ladakh. Location and Altitude: The tunnel will be constructed at an altitude of over 16,500 feet. It is situated on the Nimu-Padam-Darcha Road link, which connects Ladakh and Himachal Pradesh. The construction of this tunnel is expected to be completed by 2025. Purpose: The primary purpose of the Shinku La tunnel is to provide all-weather connectivity to the border areas of Ladakh. It will facilitate the swift movement of troops and heavy weaponry to forward areas. Significance: It will provide the Indian military access to the border areas of Ladakh from the Himachal side The completion of this project will improve the financial condition of Zanskar Valley residents by giving impetus to tourism. It will significantly enhance connectivity and security in the region. World Record: Upon completion, the Shinku La tunnel will become the longest high-altitude highway tunnel in the world. It is also referred to as the world’s highest tunnel, connecting Himachal Pradesh to Ladakh at an impressive elevation of 16,580 feet. Source: Hindustan Times FOOT ROT (BAKANE DISEASE) Syllabus Prelims – Science Context: The Punjab Agricultural University (PAU), Ludhiana, has developed biocontrol agent Trichoderma asperellum (2% WP), and registered it with the Central Insecticides Board and Registration Committee (CIBRC) Background: This registration aims to manage Foot Rot (also known as Bakane disease) in Basmati rice, which has been a persistent problem in the region, causing considerable losses to farmers and threatening the state’s export prospects. About FOOT ROT (BAKANE DISEASE): Foot Rot, also known as Bakanae Disease, is a significant fungal disease affecting rice, particularly the Basmati variety. Foot Rot is caused by the fungus Gibberella fujikuroi (also known as Fusarium monoliforme). It primarily affects the roots and basal stems of rice plants. The disease is characterized by: Abnormal elongation of seedlings Seedlings turning pale yellow Eventually, the seedlings dry up and die Impact: Foot Rot not only affects yield but also poses health risks due to mycotoxin contamination in the affected rice grains. Management: Chemical control: Historically, chemical fungicides were used to manage Foot Rot. However, their environmental impact and residue concerns have led to the search for alternative solutions. Biological control: The recent registration of Trichoderma asperellum 2% WP by Ludhiana’s Punjab Agricultural University (PAU) offers an eco-friendly approach. Trichoderma, when applied to seeds or seedlings, can suppress the growth of the Foot Rot pathogen. Cultural practices: Crop rotation, proper drainage, and avoiding waterlogged conditions can help reduce disease incidence. Source: Indian Express Constitutional Morality Syllabus Prelims- Polity Context: The recent arrests of chief ministers on corruption charges raises legal, political, and constitutional concerns and poses questions about its consistency with constitutional morality, especially in a parliamentary democracy like India. Background: Opposition has alleged a breach of legal and constitutional values. Constitutional Morality: Constitutional morality (CM) is a concept that refers to the principles and values underlying a constitution that guide the actions of both the government and the citizenry. The concept of constitutional morality was propounded by the British Classicist George Grote in the 19th century. He described CM as a “paramount reverence for the forms of the Constitution” of the land. In India, the term was first used by Dr. B.R. Ambedkar. Pillars of Constitutional Morality: Constitutional Values uphold the core values enshrined in the Constitution, such as justice, liberty, equality, fraternity, secularism, and the dignity of the individual. The Rule of Law upholds the supremacy of the law where everyone, including government officials, is subject to and accountable under the law. Democratic Principles ensure the functioning of a representative democracy where citizens have the right to participate in decision-making processes and hold their elected representatives accountable. Fundamental Rights respecting and protecting the fundamental rights guaranteed by the constitution, such as the right to equality, freedom of speech and expression, right to life and personal liberty, etc. Separation of Powers maintains the separation and balance of powers between the legislative, executive, and judicial branches of government to prevent any one branch from becoming too powerful. Checks and Balances establish mechanisms and institutions that provide checks and balances to prevent abuse of power and protect the rights of individuals. Constitutional Interpretation interprets the Constitution in a manner that promotes its underlying principles and values while adapting to changing societal needs and circumstances. Ethical Governance ensures ethical conduct in governance, transparency, accountability, and integrity in public service. Judgments Upholding Constitutional Morality: Kesavananda Bharati v. State of Kerala, 1973 established the “basic structure doctrine,” which essentially limits Parliament’s power to amend the Constitution and ensures its core principles remain intact. This can be seen as an early instance of the court upholding the spirit of the Constitution. SP Gupta Case (First Judges Case), 1982 labelled a constitutional breach as a severe violation of constitutional morality. Naz Foundation v. Government of NCT of Delhi, 2009 decriminalised consensual same-sex relationships between adults. The court emphasised that “constitutional morality” should prevail over societal perceptions of morality, upholding individual rights. Manoj Narula v. Union of India, 2014 stated that “Constitutional Morality means to bow down to the norms of the Constitution and not act in a manner which would become violative of the rule of law of action in an arbitrary manner. Indian Young Lawyers Association v. State of Kerala (Sabarimala Case), 2018 struck down the practice of excluding women of a certain age group from the Sabarimala temple. It emphasised that “constitutional morality” includes principles of justice, equality, liberty, and fraternity, which outweighed religious customs restricting women’s entry. Navtej Singh Johar v. Union of India, 2018 read down Section 377 of the Indian Penal Code, which criminalised homosexuality. Challenges to Constitutional Morality in India: One of the significant challenges is political interference in the functioning of constitutional bodies and institutions. This interference can undermine the autonomy and impartiality of these institutions, affecting their ability to uphold constitutional values. For example, there has been criticism surrounding the recent changes to the appointment committee for the Election Commission of India and the amended IT Rules 2023. Balancing judicial activism with judicial restraint is another challenge. While judicial activism can promote the protection of rights and enforcement of constitutional values, excessive activism can encroach upon the domain of the executive and legislature. Despite having a robust constitutional framework, ensuring effective enforcement and compliance remains a challenge. Implementation gaps, delays in justice delivery, and lack of awareness about constitutional rights among the general populace contribute to this challenge. Source: Indian Express Practice MCQs Daily Practice MCQs Q1.)  Consider the following statements about the Foot Rot: Foot Rot is a fungal disease affecting the Basmati rice. Crop rotation, proper drainage, and avoiding waterlogged conditions can help reduce disease incidence. Which of the statements given above is/are not correct? 1 only 2 only Both 1 and 2 Neither 1 nor 2 Q2.)The Shinku La tunnel, recently seen in news is an infrastructure project aimed at enhancing connectivity in the border areas of Arunachal Pradesh Manipur Ladakh Punjab Q3.)  With reference to the non-market economy, consider the following statements: A non-market economy refers to a country where market forces do not play a dominant role in determining prices, wages, and resource allocation. Non-market economies may have restrictions on currency convertibility. Non-market economies are subject to anti-dumping duties imposed by other countries. How many of the statements given above are correct? Only one Only two All three None Comment the answers to the above questions in the comment section below!! ANSWERS FOR ’  11th May 2024 – Daily Practice MCQs’ will be updated along with tomorrow’s Daily Current Affairs.st ANSWERS FOR  10th May – Daily Practice MCQs Answers- Daily Practice MCQs Q.1) – d Q.2) – b Q.3) – c

Daily Prelims CA Quiz

UPSC Quiz – 2024 : IASbaba’s Daily Current Affairs Quiz 11th May 2024

The Current Affairs questions are based on sources like ‘The Hindu’, ‘Indian Express’ and ‘PIB’, which are very important sources for UPSC Prelims Exam. The questions are focused on both the concepts and facts. The topics covered here are generally different from what is being covered under ‘Daily Current Affairs/Daily News Analysis (DNA) and Daily Static Quiz’ to avoid duplication. The questions would be published from Monday to Saturday before 2 PM. One should not spend more than 10 minutes on this initiative. Gear up and Make the Best Use of this initiative. Do remember that, “the difference between Ordinary and EXTRA-Ordinary is PRACTICE!!” Important Note: Don’t forget to post your marks in the comment section. Also, let us know if you enjoyed today’s test 🙂After completing the 5 questions, click on ‘View Questions’ to check your score, time taken, and solutions.To take the Test Click Here

Daily Prelims CA Quiz

UPSC Quiz – 2024 : IASbaba’s Daily Current Affairs Quiz 10th May 2024

The Current Affairs questions are based on sources like ‘The Hindu’, ‘Indian Express’ and ‘PIB’, which are very important sources for UPSC Prelims Exam. The questions are focused on both the concepts and facts. The topics covered here are generally different from what is being covered under ‘Daily Current Affairs/Daily News Analysis (DNA) and Daily Static Quiz’ to avoid duplication. The questions would be published from Monday to Saturday before 2 PM. One should not spend more than 10 minutes on this initiative. Gear up and Make the Best Use of this initiative. Do remember that, “the difference between Ordinary and EXTRA-Ordinary is PRACTICE!!” Important Note: Don’t forget to post your marks in the comment section. Also, let us know if you enjoyed today’s test 🙂After completing the 5 questions, click on ‘View Questions’ to check your score, time taken, and solutions.To take the Test Click Here